+ All Categories
Home > Documents > Antitrust McChesney

Antitrust McChesney

Date post: 27-Apr-2015
Category:
Upload: throwaway1440
View: 189 times
Download: 2 times
Share this document with a friend
84
ANTITRUST OUTLINE ANTITRUST OUTLINE I. INTRODUCTION AND OVERVIEW A. THE LAW AND ECONOMICS OF ANTITRUST 1. Introduction: Antitrust promotes competition out of the belief that competition presses producers to satisfy consumer wants at the lowest price while using the fewest resources. Producer rivalry lets consumers bid for goods and services, thus matching their desires with society’s opportunity costs. 2. Some Basic Explanations and Behavioral Assumptions: Antitrust tries to assure that the gap between the ideal of competition and the reality of some forms of private rule does not grow dangerously wide. a. The Demand Schedule: When economists refer to “demand,” they are identifying a demand schedule— a statement of the different quantities of a good or service that a consumer would buy at each of several different price levels. Because the amount of an item that a person will purchase cannot be determined without also considering its price, demand cannot be identified as a set, specific quantity. Rather, the demand for a product consists of a range of alternative quantities. The basic rule is that the value a consumer will attach to successive units of a commodity diminishes as her total consumption of that commodity increases. In other words, the quantity demanded varies inversely with price; the demand curve is negatively or downwardly sloped. b. Profit-Maximizing Behavior by Firms: In making production decisions, the firm will adhere to the principle of substitution—that for a given set of technical possibilities, efficient (profit- maximizing) production will substitute cheaper factors for more expensive ones. Efficient production generally means that a firm will seek 1
Transcript
Page 1: Antitrust McChesney

ANTITRUST OUTLINEANTITRUST OUTLINE

I. INTRODUCTION AND OVERVIEW

A. THE LAW AND ECONOMICS OF ANTITRUST

1. Introduction: Antitrust promotes competition out of the belief that competition presses producers to satisfy consumer wants at the lowest price while using the fewest resources. Producer rivalry lets consumers bid for goods and services, thus matching their desires with society’s opportunity costs.

2. Some Basic Explanations and Behavioral Assumptions: Antitrust tries to assure that the gap between the ideal of competition and the reality of some forms of private rule does not grow dangerously wide.a. The Demand Schedule: When economists refer to “demand,” they are

identifying a demand schedule—a statement of the different quantities of a good or service that a consumer would buy at each of several different price levels. Because the amount of an item that a person will purchase cannot be determined without also considering its price, demand cannot be identified as a set, specific quantity. Rather, the demand for a product consists of a range of alternative quantities. The basic rule is that the value a consumer will attach to successive units of a commodity diminishes as her total consumption of that commodity increases. In other words, the quantity demanded varies inversely with price; the demand curve is negatively or downwardly sloped.

b. Profit-Maximizing Behavior by Firms: In making production decisions, the firm will adhere to the principle of substitution—that for a given set of technical possibilities, efficient (profit-maximizing) production will substitute cheaper factors for more expensive ones. Efficient production generally means that a firm will seek the lowest possible costs for a particular rate of output. A profit-maximizing firm will increase production when the additional revenue exceeds the additional costs. That is, the firm will expand its output as long as the marginal, or last, unit add more to revenues than it does to costs—namely, as long as the marginal revenue exceeds or equals marginal cost.

P

p E (Profit Maximization)

q Q3. Basic Economic Models: Economic theory traditionally concludes that the

structure of an industry affects its behavior and, ultimately, its performance.

1

MR (marginal revenue)

MC (marginal cost)

Page 2: Antitrust McChesney

a. Perfect Competition: Perfect competition describes a market where consumer interests are controlling. The market is efficient in the sense that no rearrangement of production or distribution will improve the position of any consumer or seller. Societal wealth is maximized because resources are put to their highest valued use and output is optimal. The following conditions are useful in predicting whether competitive behavior is likely in a market: (1) there are many buyers and sellers; (2) the quantity of the market’s products bought by any buyer or sold by any seller is so small relative to the total quantity traded that changes in these quantities leave market prices unaffected; (3) the product is homogenous; no buyer has a reason to prefer a particular seller and vice versa; (4) all buyers and sellers have perfect information about market prices and the nature of the goods sold; (5) there is complete freedom of entry into and exit out of the market.

PMC

(Output of a Competitive Industry)

DQ

In a perfectly competitive market, the individual firm is merely a quantity adjuster. All firms sell at marginal cost and earn only a normal return on investment. Each firm takes a price as set by the market; no firm can affect the price by adjusting output by raising or lowering price. Each firm strives to maximize profits by adjusting its output until its marginal costs equals the prevailing market price.

b. Monopoly: A seller with monopoly power restricts her output in order to raise her price and maximize her profits. Not only does this transfer output and may relieve the producer of pressure to innovate or otherwise be efficient. Monopoly markets are often described by three structural and functional factors: (1) one seller occupies the entire market; (2) the seller’s product is unique (i.e. there are no substitutes); (3) substantial barriers bar entry by other firms into the industry, exit is difficult. For a competitive seller marginal revenue is the same at all output levels; the monopolist, on the other hand, finds marginal revenue always less than price because her demand curve slopes downward. As a result, the monopolist faces a choice on production and price: either sell at a higher price (with fewer unit sales) or sell at a lower price (with greater unit sales). In making this choice, the monopolist will maximize profits at less than the competitive output level—namely, where marginal revenue equals marginal cost. Thus, contrary to the competitive result, the

2

Page 3: Antitrust McChesney

monopolist will maximize profits by restricting output and setting price above marginal cost.

MCP

(Profit Maximizing Monopolist)

D

QMR

The monopolist maximizes her profit by producing an output quantity where her marginal revenue equals marginal cost and by charging whatever price her demand curve reveals is necessary to sell that output. In other words. The monopolist will increase her output only as long as her profits increase.

c. Competition and Monopoly Compared: Compared to perfect competition, the primary effects of monopoly are reduced output, higher prices, and a transfer of income from consumers to producers.

P

(Monopolizing a Competitive Industry)

D

Q

MRMonopoly pricing also leads to what is known as deadweight welfare loss. Deadweight loss represents the loss in value to consumers who at the competitive price would buy product, but who at the monopoly price are deflected to “inferior” substitutes.

d. Oligopoly: Where the market contains only a few sellers, all sellers recognize that they are largely interdependent. Therefore, each seller accounts for its rivals’ reactions when setting output and prices. This means that oligopolists will not drop prices to increase market share because they expect that any gains will be cancelled immediately when rival sellers retaliate with similar price cuts. As a consequence,

3

pm

qm

pm

pc

qm qc

Deadweight loss

Income transfer

MC

Page 4: Antitrust McChesney

oligopoly sellers focus on coordination and anticipation. This leads to cartelization. The problem here, however, is that cartels are unstable, and each member has the incentive to “chisel” profits away from the other members of the cartel.

4. Overview of Antitrust Statutes: The common law’s inability to reach certain anticompetitive behavior and rising concern over abuses by corporate giants in the late nineteenth century spawned legislation curbing the power of the railroads and “trusts.”a. The Sherman Act: The Sherman Act, enacted in 1890, is the principal

antitrust statute. Plaintiffs prevailing under the Sherman Act are entitled to treble damages, costs, and attorney’s fees.i. Section One: Section one of the Sherman act make unlawful (and

criminal) “every contract, combination or conspiracy in restraint of trade” in interstate or foreign commerce. Anything traditionally considered a contract qualifies, but even less traditional notions may be enough. Supreme Court decisions have limited this language, interpreting it to prohibit only “unreasonable” restraints of trade.

ii. Section Two: Section two prohibits monopolizing, attempts to monopolize, and combinations or conspiracies to monopolize any part of interstate or foreign commerce. Section two is typically sued to attack the activity of a single firm with monopoly power.

b. The Clayton Act. Enacted in 1914, the Clayton Act was a response to perceived deficiencies or loopholes in the Sherman Act.i. Section 3: Section three prohibits sales on the condition that the

buyer not deal with competitors of the seller (“tying” and “exclusive dealing”), where the effect “may be substantially to lessen competition or tend to create a monopoly.”

ii. Section 4: Allows private parties injured by violations of the Sherman and Clayton Acts to sue for treble damages.

iii. Section 7: Prohibits acquisitions or mergers where the effect “may be substantially to lessen competition, or tend to create a monopoly.”

c. Federal Trade Commission Act: Also enacted in 1914, the Act created the Federal Trade Commission, giving it broad powers to enforce the antitrust laws. Section 5(a)(1) prohibits “unfair methods of competition in or affecting commerce, and unfair or deceptive acts or practices in or affecting commerce.”

B. OVERVIEW OF ANTITRUST IN THE COURTS

1. Monopolization: Section 2 of the Sherman Act does not specifically outlaw monopolies as such, but rather, it forbids the act of monopolizing. Thus, the possession of monopoly power and the willful acquisition or maintenance of that power, will violate § 2.a. Aspen Skiing Co. v. Aspen Highlands Skiing Corp. (1985): The Court

unanimously held that a monopolist’s refusal to continue to participate in a joint marketing plan with its only rival could amount to monopolization. The Court distinguished between parties with a history

4

Page 5: Antitrust McChesney

of dealing and those with no history of dealing. Thus, the refusal by the owner of three of four mountain slopes used for skiing in Aspens to cooperate with the owner of the fourth slope and continue their joint ski ticket reflected “a deliberate effort to discourage customers from doing business with its smaller rival.” The “decision by a monopolist to make an important change in the character of the market,” coupled with the jury’s conclusion that the defendant lacked a reasonable business justification, supported the conclusion that the behavior was unlawful.i. Market Definition: The key to this case was the definition of the

relevant market—i.e., whether the defendant had enough “market power” to make it a monopolist. The market was defined as downhill skiing in Aspen, Colorado. Defendant would have preferred to define the market more broadly, to include other ski resorts in the area.

ii. Refusal To Deal: A company that has monopoly power has no duty to cooperate with its business rivals and does not violate § 2 by refusing to deal with competitors if it has valid business reasons for the refusal. If the rivals have history of dealing, however, the monopolist might act unlawfully if it “changes the character of the market” by refusing to continue dealing.

iii. Essential Facilities Case: The Court describes this as an essential facilities case—i.e. the multi-ticket is essential to the business. However, the Court ignores the essential facilities analysis but focuses instead on the legitimacy of the defendant’s reasons for refusing to agree with the plaintiff.

b. Implications: Aspen’s implication that a monopolist could violation § 2 merely by changing its distribution pattern and, indeed, that the monopolist can be required to cooperate with its competitors in a joint marketing arrangement is disturbing. Perhaps most important, such a standard raises the risks associated with undertaking any form of legitimate collaboration with a direct rival. By raising the potential costs of abandoning such relationships, Aspen makes it less likely that collaborative arrangement will be formed in the first place.

2 Vertical Restraints: Relationships between the various levels of production and distribution can be described as “vertical”; on the other hand, relationships between competitors are described as “horizontal.a. Graphic Products Distributors, Inc. v. Itek Corp. (11th Cir. 1983): This

case involved nonprice, vertical restraints on trade. This was basically a territorial allocation case, in which the plaintiff claimed that the manufacturer’s use of exclusive territories was an unreasonable restraint of trade. The court noted that the Supreme Court had prescribed a rule of reason analysis for such claims. See Sylvania. The Court noted that while vertical restraints may lessen intrabrand competition, it may enhance interbrand competition. Thus, the question in this case was what were the competitive effects of the vertical territorial allocation. The first factor to be examined in answering that question is whether the defendant has market power, as evidenced by market share. (The

5

Page 6: Antitrust McChesney

defendant in this case never challenged the definition of the relevant product market, a major tactical error.) Once market power has been demonstrated, the plaintiff must show an anticompetitive effect, either in the intrabrand or interbrand markets. That anticompetitive effect must be balanced against any alleged positive effects on interbrand competition stemming from the restraints. The ultimate question is, what effect does the restraint have on consumers?

b. Copperweld Corp. v. Independence Tube Corp. (1984): The Court overruled the intra-enterprise conspiracy doctrine and held that a parent and its wholly owned subsidiary could not be “conspiring entities” for the purpose of satisfying § 1’s plurality requirement. Antitrust is about economics,; it is not about formalities.

3. Conspiracy to Restrain Trade: A primary concern of the antitrust laws has been to preserve and encourage competition among firms in the same industry. The antitrust laws place limits on collaboration among competing firms, so called horizontal restraints. When a group of competitors agree not to deal with a firm outside the group, there is a combination in restraint of trade.a. Rothery Storage & Van Co. v. Atlas Van Lines (D.C. Cir. 1986): This is

a horizontal boycott which, under Supreme Court precedent, are per se illegal. Judge Bork, however, notes that most lower courts do not follow the rule that any per se horizontal restraint is per se illegal. Atlas, a national moving company with interstate authority, operated its business though various agent contracts, in which it delegated its authority to transport goods interstate to smaller companies that would do the actual work. When the moving industry was deregulated in 1979, the agents had authority to move interstate, and thus operated as both as an agent and a competitor of Atlas. This created a free riding problem for Atlas: “A free ride occurs when one party to an arrangement reaps benefits for which another party pays, though that transfer of wealth is not part of the agreement between them.” Basically, the agents were using Atlas equipment, uniforms, and services on non-Atlas interstate shipments, thus consuming Atlas goods and exposing Atlas to the potential for liability. To address the problem, Atlas announced that it would cancel the contract of any agent that continued to handle interstate carriage on its own account as well as for Atlas.i. Rejection of Per Se Analysis: Judge Bork concluded that “the

challenged restraint is ancillary to the economic integration of Atlas and its agents so that the rule of per se illegality does not apply. Neither are the other tests of the rule of reason offended since Atlas’ market share is far too small for the restraint to threaten competition or to have been intended to do so.”

ii. Relevant Market: Bork defines the relevant market by looking to the “availability of substitutes.” “The degree to which a similar product will be substituted for the product in question is said to measure the cross-elasticity of demand, while the capability of other production facilities to be converted to produce a substitutable product is referred to as the cross-elasticity of

6

Page 7: Antitrust McChesney

supply. The higher these cross elasticities, the more likely it is that similar products or the capacity of production facilities now used for other purposes are to counted in the relevant market.”

iii. Economic Reality: Bork’s analysis focuses not on the formalities of prior precedent, but on the economic realities of Atlas’ cancellation of the contracts. “If it is clear that Atlas and its agents by eliminating competition among themselves are not attempting to restrict industry output, then their agreement must be designed to make the conduct of their business more effective.” Based on the fact that Atlas was seeking to eliminate free riding, “the Atlas agreements thus produce none of the evils of monopoly but enhance consumer welfare by creating efficiency.

iv. Ancillary Restrictions: Bork notes that every combination “restrains” trade or reduce competition; but every combination between competitors cannot be per se illegal under the Sherman Act. Quoting Justice (then Judge) Taft in United States v. Addyston Pipe & Steel Co. (1898), Bork states that “’restrictions in the articles of partnership upon the business activity of the members, with a view of securing their entire effort in the common enterprise, were, of course, only ancillary to the main end of the union, and were to be encouraged.’ To be ancillary, and hence exempt from the per se rule, an agreement eliminating competition must be subordinate and collateral to a separate, legitimate transaction.”

b. BMI, NCAA, and Pacific Stationary: Bork relies on these seminal Supreme Court cases of horizontal restrictions to support his decision that only restraints that so tend to restrict competition that no procomptetative justification can be discerned are subject to the per se rule.

4. Injury to Competition Through Mergers: It is difficult to overstate the importance of antimerger policies to the American antitrust system. This concern is most acute where the participants are direct rivals, for courts often presume that such arrangements are more prone to restrict output and increase prices. The economic analysis indicates that antirust law has a legitimate concern with preservation of market structure that is conducive to effective competition.a. United States v. Waste Management, Inc. (2nd Cir. 1984): This was a

merger challenged under § 7 of the Clayton Act. The case illustrates that market share might not be as dispositive as it seems. A large market share resulting from a merger is prima facie evidence of illegality; easy of entry, however, may rebut the presumption of illegality because it demonstrates that the firm will be unable to raise prices over the competitive level without new firms entering the market. (Judge Winters suggests that potential entrants to the market might be considered in determining market power; but that analysis can just as easily be done by looking at the rebuttal case.) The court concluded that “entry into the

7

Page 8: Antitrust McChesney

relevant product and geographic market by new firms or by existing firms [outside the geographic market] is so easy that any anti-competitive impact of the merger would be eliminated more quickly by such competition than by litigation.”

b. United States v. Syufy Enterprise (9th Cir. 1990): The court followed the logic in Waste Management, and refused to invalidate the merger of the only two firms in the market—i.e., a “merger to monopoly.”

5. Balancing Types of Error In Antitrust: Because courts don’t always make the right decision, they should be aware of what type of “error” might be possible as the result of an erroneous decision: one possible error is finding illegal economically benign actions that perhaps ought positively to be encouraged; the second possible error is finding no illegality for practices that are in fact economically malign and ought to be deterred. These types of error are often referred to as Type I (a “false positive,” analogous to mistakenly punishing the innocent) and Type II (a “false positive,” exemplified by mistakenly failing to punish the guilty).a. Entry: As the Waste Management decision illustrated, when barriers to

entry are low, antitrust errors tend to be self-correcting. As quantities are reduced and prices rise, new entrants emerge to redress any difficulty. The costs of Type II error thus are reduced.

b. The Costs of Type I Error: Type I error is not the subject of any significant self-canceling mechanism. When a court imposes liability on conduct that actually is beneficial, there is no market corrective for the judicial mistake; society loses the benefits of the commercial practice that has been wrongly deterred. Only reversal of the case can undo the mistake.

6. Special Requirements for Private Recovery: Most antitrust suits are brought by private plaintiffs. In the 1970s, the Supreme Court established two concepts that serve as a barrier to private antitrust suits—antitrust standing, and antitrust injury.a. Mid-Michigan Radiology Assocs. v. Central Michigan Community

Hosp. (E.D. Mich. 1995): The court defines antitrust injury as an injury resulting from a violation of “the type of interests protected by the antitrust laws.” In other words, was the injury to competition itself, or merely to a competitor: antitrust injury flows from a reduction in competition, not from competition itself. Antitrust standing asks whether the injury to the plaintiff was caused by the antitrust injury; in other words, is the plaintiff an efficient enforcer of the antitrust laws. To determine standing, it is necessary to examine the physical and economic nexus between the alleged violation and the harm to the plaintiff. The plaintiff in this case did not show antitrust injury, and lacked standing. Its complaint was that the defendant cancelled its contract, not that the replacement contract was anticompetitive.

b. Indirect Purchasers: Note that in Illinois Brick, the Supreme Court held that indirect purchasers of a price-fixed product do not have standing to bring an antitrust suit; only direct purchasers have the requisite proximity of injury.

8

Page 9: Antitrust McChesney

II. CONSPIRACIES IN RESTRAINT OF TRADE

A. THE MECHANICS OF PRICE-FIXING ARRANGEMENTS

1. How Price Fixing Works: The International Uranium Cartel: When the uranium market declined in the 1950s and 1960s, the U.S. government shut out foreign producers from the U.S. market. By the 1970s, the industry was “overbuilt,” with prices barely covering the variable operating costs. Foreign producers responded by creating a cartel to divide up the rest of the world’s market. The cartel set prices, allocated sales by nation group and by producer within each group, and provided a rigged bidding system in which one “leader” quoted the fixed price, while other companies quoted higher prices in order to create the appearance of competition. The producers set a permanent Secretariat whose function was to administer the cartel’s activities, and an Operating Committee which met regularly to set the prices for individual bids.

2. Normative Pros and Cons of Cartels: Per se bans on price fixing assume that price agreements have no redeeming social value under any circumstances. Economists recognize situations in which cartels can be beneficial.a. Ruinous Competition: In industries with high fixed costs but low

marginal costs, price wars can keep competitors from being able to maintain their facilities in the long run. It is possible to imagine a situation under which collusion is necessary for achieving competitive outcomes. However, antitrust has yet to condone price fixing to solve the fixed-cost, “ruinous competition” problem.

b. Self-Help Remedies in Contract: Sometimes, collective action is an effective, extra-judicial way of helping firms enforce their contracts. This often takes the form of information exchanges. In at least one case, the Supreme Court refused to enjoin an information exchange because “the gathering and dissemination of information which will enable sellers to prevent the perpetration of fraud upon them . . . cannot be held to be an unlawful restraint of trade.” Cement Manufacturers Protective Ass’n v. United States.

c. Lower Prices Through Collusion: In certain situations, fierce competition may destroy profitability in the market. In that situation, sellers may agree to fix prices at a “reasonable” level, i.e. one that gave them a competitive but not a supra-competitive profit. Economically, however, this type of an agreement is unlikely because new firms will enter and drive prices down, or existing firms will lower prices as the profit risks decline.

d. Property Rights and Establishment of Efficient Prices: Defendants often claim that price fixing is necessary to establish the “right” price, arguing in effect that the market did not price at efficient level without collusion. Ruinous competition is a subset of this argument.

e. Enforcement of Rules Against Price Fixing: The nirvana fallacy states that just because something undesirable is going on does not mean that passing a law against it will obviate the problem. Having no law may engender Type II errors; but having a law may create Type I errors. In the end, the issue is an empirical one: does the law find and punish the

9

Page 10: Antitrust McChesney

anticompetitive behavior, while leaving the unobjectionable behavior alone?

B. CLASSIC EARLY CASES

1. United States v. Trans-Missouri Freight Ass’n (1897): This was an association of railroad carriers that set freight rates for all participants. They conceded that their agreement curbed each firm’s commercial freedom, but that, as regulated industries, they were exempt from the Sherman Act, and that in any case the rates they fixed were reasonable. The Court held that compliance with another statute (e.g. regulatory restrictions) does not exempt a firm from the antitrust laws; and furthermore, the “reasonableness” of prices fixed pursuant to a contract was irrelevant under the Act—Congress intended for the Court to create a new federal common law of antitrust.a. Two Principles: This cases thus established two things: first, that price

fixing was per se illegal under § 1 of the Sherman Act; and second, that the solution to ruinous competition was not price fixing agreements. “Competition itself brings the charges down to what may be reasonable, while, in the case of an agreement to keep prices up, competition is allowed no play.”

b. United States v. Joint Traffic Ass’n (1898): The Court backed off the strict language in Trans-Missouri, and held that if the price agreement was “collateral” to the main price agreement it is not a violation of antitrust law.

2. United States v. Addyston Pipe & Steel Co. (6th Cir. 1898): Six cast iron pipe producers conceded that they had agreed to divide the southern and western markets into regional monopolies and had fixed prices for each territory. The defendants argued that the agreement was designed to eliminate ruinous competition, and that the fixed prices were reasonable. Judge Taft held that the agreements deprived the public of the benefits of competition. Seeking to build an airtight per se ban against price-fixing agreements, Taft read the prevailing common law as voiding all price-fixing agreements unless they were ancillary to some legitimate cause. “If the sole object of both parties in making the contract as expressed therein is merely to restrain competition, and enhance or maintain prices, it would seem that there was nothing to justify or excuse the restraint . . . and therefore would be void.” Thus, “naked” restraints in which the “sole object” is to eliminate competition are per se illegal; ancillary price-fixing agreements may be legal if they are reasonable.

3. United States v. Trenton Potteries Co. (1927): The makers of 82 percent of toilets and other bathroom fixtures belonged to an association that had fixed the prices of sanitary pottery and had limited sales to “legitimate jobbers.” The Court again rejected the “reasonableness” of prices as a defense to price fixing agreements under § 1: “the reasonable price fixed today may through economic and business changes become the unreasonable price tomorrow. . . . Whether prices actually agreed upon were reasonable is immaterial . . . .” Read literally, this analysis holds that proof of the mere existence of a price-fixing agreement establishes defendant’s illegal purpose, and the prosecution need not show that the prices fixed are unreasonable. On the other hand, the defendants’ control of 82 percent of the market evidenced significant market power.

10

Page 11: Antitrust McChesney

4. Appalachian Coals, Inc. v. United States (1933): To cope with plunging prices, 137 coal companies, which accounted for roughly 12 percent of all bituminous coal production east of the Mississippi, formed a new company to act as the exclusive selling agent for member firms. The agency was instructed to get the “best prices obtainable” and if all output could not be sold, to allocate orders fairly among member firms. In effect, it served as a sales cartel, but with far from complete control over the coal market. The Court refused to reject the agreement, and held that because the agreement did not fix prices it was not per se illegal: “Realities must dominate the judgment. The mere fact that the parties to an agreement eliminate competition between themselves is not enough to condemn it.” The Court accepted that the collaboration was for legitimate ends.

C. DOCTRINAL FOUNDATIONS OF § 11. Per Se Violations: Certain agreements almost always result in substantial

restraint of trade, without redeeming procompetitive benefits. A case-by-case inquiry into reasonableness is therefore unwarranted. Consequently, certain types of business agreements among competitors are unreasonable as a matter of law—i.e., illegal per se.a. United States v. Socony-Vacuum Oil Co. [“Madison Oil”] (1940): The

facts strongly resembled those of Appalachian Coals. The oil refining industry was depressed, and independent producers faced panic market conditions. Independent refiners lacked storage facilities and had been dumping gasoline at give-away prices. In response, a group of major refining companies agreed to buy surplus (“distress”) gasoline from the independents, disposing of it in a more orderly manner so as not to depress prices. The arrangement assigned a major firm to each independent as a “dancing partner,” and the major firm bought its partner’s surplus gasoline.i. Holding: Relying on Trenton Potteries, the Court held that

“uniform price-fixing by those controlling in any substantial manner a trade or business in interstate commerce is prohibited by the Sherman Law, despite the reasonableness of the particular prices agreed upon.” This resolved the conflict between Appalachian Coals and Trenton Potteries. The Court believed that the defendants, by manipulating the relatively thin spot market, kept gasoline prices above the level that competition would otherwise have yielded. Even this type of “price stabilization” is per se illegal under § 1. With some exceptions, Socony-Vacuum remains a foundation for analysis of horizontal price-fixing cartels today.

ii. Rationale: Although the defendants had not explicitly agreed on the price at which they would sell their gasoline, the Court found that the arrangement’s purpose was to curtail competition and raise prices. “Any combination which tampers with price structures is engaged in an unlawful activity. Even though the members of the price-fixing group were in no position to control the market, to the extent that they raised, lowered, or stabilized prices they would be directly interfering with the free play of

11

Page 12: Antitrust McChesney

market forces.” “Under the Sherman Act a combination formed for the purpose and with the effect of raising, depressing, fixing, pegging, or stabilizing the price of a commodity in interstate commerce is illegal per se.”

b. Fashion Originators’ Guild of America v. FTC (1941): Women’s garment manufacturers who claimed to be creators of original dress designs sought to curb “style piracy” by which other manufacturers copied their designs and sold these copies at much lower prices. To stop the practice the Guild’s member agreed to refuse to sell to retailers who also sold garments copied from a Guild member. The members were trying to prevent an allegedly illegal or tortious act; however, the Guild was taking the law into its own hand and, in the process, excluding rivals from the market.i. Holding: The Court held that because the combination “narrows

the outlets to which garment and textile manufacturers can sell and the sources from which retailers can buy; subjects all retailers and manufacturers who decline to comply to an organized boycott; takes away the freedom of action of members by requiring each to reveal to the Guild the intimate details of their individual affairs and has both as its necessary tendency and as its purpose and effect the direct suppression of competition from the sale of unregistered textiles and copied designs,” the combination violated § 1.

ii. Effect: FOGA has been read as applying a per se test to group boycotts. The Court invoked the notion that the antitrust laws were designed to protect “small dealers and worthy men.” The holding in this case is, however, in tension with the holding in Cement Manufacturers.

c. United States v. Topco Associates, Inc. (1972): A group of small- and medium-sized grocery chains with 6 percent of the market created a joint subsidiary to market private label products through their stores in competition with larger supermarket chains. The members of the joint venture allocated exclusive territories in which to sell the new label, and refused to allow new members whose location would overlap with the territory of an existing member. This is a horizontal combination because the members of the joint venture were competitors; but it was argued that the combination removed the incentive for these stores to compete.i. Holding: The Court held that market allocations are per se illegal

whether or not ancillary to price-fixing or other market-rigging arrangements. “The Court has reiterated time and time against the horizontal territorial limitations are naked restraints of trade with no purpose except stifling of competition.”

ii. Rationale: In reaching this result and applying a per se rule, the Court specifically rejected the fact that the defendants lacked market power, that the arrangement did not reduce competition among them or in the market, and that the restriction were

12

Page 13: Antitrust McChesney

necessary for the joint venture to succeed. The Court said these arguments were for Congress to assess and observed that courts are ill equipped to measure whether restraints on competition in one area are overcome by increased competition elsewhere.

iii. Criticism: The Court’s opinion is subject to criticism for (1) failing to evaluate the economic necessity and competitive benefits of the arrangements, and (2) ignoring that the participants lacked market power and therefore could not have affected competition adversely.

2. Rule of Reason: An all-inclusive condemnation of every restraint, if applied literally, would make § 1 unworkable since it would condemn much business behavior that is economically and socially beneficial. Thus, certain combinations are only condemned if they are unreasonable—in other words, court will consider whether the restrain promotes competition more than it suppresses competition.a. Chicago Board of Trade v. United States (1918): The government

contended that a grain exchange rule requiring members to adhere to their closing bid on the “call” (which in effect confined price competition to the time the exchange was open) was illegal because it fixed prices during part of the business day. The Court rejected the claim of per se illegality, and stated that “the true test of legality is whether the restraint imposed is such as merely regulates and perhaps thereby promotes competition or whether it is such as may suppress or even destroy competition.”i. Relevant Factors: To measure and evaluate a challenged

restraint’s impact on competition, courts should consider “the facts peculiar to the business to which the restrain is applied; its condition before and after the restraint was imposed; the nature of the restraint and its effect, actual or probable. The history of the restraint, the evil believed to exist, the reason for adopting the particular remedy, the purpose or end sought to be attained, are all relevant facts. This is not because a good intention will save an otherwise objectionable regulation or the reverse; but because knowledge of intent may help the court to interpret facts and to predict consequences.”

ii. Problems: The problems with the approach created in Chicago Board of Trade was that the decision offered little guidance about how the factors should be considered in certain cases. This difficulty in administration led courts to expand the category of conduct denominated illegal per se.

b. Nat’l Soc’y of Professional Engineers v. United States (1978): The Society had a rule against competitive bidding because it was thought that “bidding on engineering services is inherently imprecise, would lead to deceptively low bids, and would thereby tempt individual engineers to do inferior work with consequent risk to public safety and health.” The Court held that the rule of reason applied, in part because the industry was a “learned profession,” and under prior decisions the Court treated

13

Page 14: Antitrust McChesney

restraints in such industries under the rule of reason. Nevertheless, the Court found the ban on competitive bidding to be an unreasonable restraint under the rule of reason. The Court said that the agreement suppressed and destroyed competition, and had an indirect (and perhaps intentional) effect on prices.i. Two Categories: The Court stated that there are “two

complementary categories of antitrust analysis. In the first category are agreements whose nature and necessary effect are so plainly anticompetitive that no elaborate study of the industry is need to establish their illegality—they are illegal per se. in the second category are agreements whose competitive effect can only be evaluated by analyzing the facts peculiar to the business, the history of the restrain, and the reasons why it was imposed.”

ii. Holding: The Court held that the Society’s “ban on competitive bidding prevents all customers from making price comparisons in the initial selection of an engineer, and imposes the Society’s views of the costs and benefits of competition on the entire market.” The holding in this case demonstrates that rule of reason analysis only looks to the effects that the challenged activity has on competition.

c. California Soc’y of Dentists v. FTC (1999): The FTC went after California dentists who enacted a rule against members advertising discounted prices and making quality claims. The FTC saw this as a type of price fixing. The Court evaluated the restraint under the rule of reason because dentists are professionals. The dentists argued that the restriction was enacted because consumers are ignorant and unable to adequately evaluate prospective services. The Court agreed with the dentists’ justification as an adequate reason to restrict competition.

D. DOCTRINAL REFORMULATIONS

1. Loosening of Per Se Rules: The Professional Engineers decision demonstrated that the Court might be willing to narrow the scope of per se illegality, while more readily finding agreements not covered by the per se rule illegal under the rule of reason.a. Price Fixing: In two of the seminal modern antitrust opinions, the Court

applied rule of reason analysis to what were essentially price fixing cases, recognizing that even price-fixing agreements may serve necessary and beneficial purposes.i. Broadcast Music, Inc. v. Columbia Broadcasting Sys. (1979)

(BMI): This case involved non-profit organizations which enforced the copyright rights of composers whose music was being publicly performed. BMI and ASCAP granted blanket licenses to radio stations, etc., and paid out royalties to the artists based on the number of times their songs were played—essentially, they were license clearing houses for their members. The blanket license was non-exclusive; thus, consumers could still buy individual licenses direct from the copyright owners. CBS approached BMI and ASCAP and asked for a license on a

14

Page 15: Antitrust McChesney

per use basis. When they refused, CBS sued for violation of the Sherman Act, including a claim of price fixing. On its face the blanket license seemed to transgress Socony-Vacuum’s sweeping ban on all contracts having the purpose or effect of “raising, depressing, fixing, pegging, or stabilizing prices.”(a). Holding: The sole question before the Court was whether

the blanket license was per se illegal. The Court held that the blanket license was not a “naked restraint on trade with no purpose except stifling competition, but rather accompanies the integration of sales, monitoring, and enforcement against unauthorized copyright use.” “Not all arrangements among actual or potential competitors that have an impact on price are per se violations of the Sherman Act or even unreasonable restraints.”

(b). Property Rights: The Court found it important that the arrangement was helping artists enforce their copyrights, which they would have been unable to do otherwise. “The extraordinary number of users, the ease with which a performance may be broadcast, the sheer volume of copyrighted compositions, the impracticality of negotiating individual licenses for each composition, all combine to create unique market conditions for performance rights.” This basically revived Addyston Pipe, and the notion that price fixing is justified if it is ancillary to a more noble purpose.

ii. NCAA v. University of Oklahoma (1984): The Court applied rule of reason analysis to another price fixing agreement; this time, however, finding a violation of § 1. The agreement was among members of the NCAA to restrict how often each team’s football games could be televised. The Court applied rule of reason analysis because the “horizontal restraints on competition are essential if the product [league sports] is to be available at all.” “BMI squarely hold that a joint selling arrangement may be so efficient that it will increase sellers’ aggregate output and thus be procompetitive. . . . Thus, despite the fact that this case involves restraints on the ability of member institutions to compete in terms of price and output, a fair evaluation of their competitive character requires consideration of the NCAA’s justifications for the restraints.” The Court noted that, because the agreement was a restriction on both price and output, it was a “naked restraint” on competition, and required some competitive justification”—in other words, was the exclusive, joint marketing arrangement necessary for college football to exist? The answer to that question was “no”; the agreement had none of the redeeming qualities of BMI.

15

Page 16: Antitrust McChesney

b. Concerted Refusals to Deal: Over the past decade, some Supreme Court decisions have retreated from an unqualified rule of per se illegality for concerted horizontal refusals to deal. C.f. FOGA.i. Northwest Wholesale Stationers v. Pacific Stationary (1985): A

seller of office supplies was expelled from a purchasing cooperative after it expanded its operations from retailing to include wholesale activities. The reason for the expulsion was disputed, and the district court granted the defendant summary judgment under the rule of reason because there was no evidence of anticompetitive effect. Justice Brennan noted that the cooperative raised efficiency by realizing scale economies in both buying and warehousing supplies, enabling smaller retailers to reduce prices and compete more efficiently. Thus, “the act of expulsion from a wholesale cooperative does not necessarily imply anticompetitive animus and thereby raise a probability of anticompetitive effect.” “Unless the cooperative possesses market power or exclusive access to an element essential to effective competition, the conclusion that expulsion is virtually always likely to have anticompetitive effect is not warranted. Absent such a showing with respect to a cooperative buying arrangement, courts should apply a rule of reason analysis.” This decision was a genuine improvement over the Court’s earlier boycott decisions and their suggestion that § 1 forbade all concerted horizontal refusals to deal.

ii. FTC v. Indiana Federation of Dentists (1986): The members of the dental federation refused to submit dental x-rays to patients’ insurers that requested the patient x-rays to evaluation the necessity of the treatment. The Court used a rule of reason approach, explaining that “the per se approach has generally been limited to cases in which firms with market power boycott suppliers or customers in order to discourage them from doing business with a competitor—a situation obviously not present here.” Nevertheless, the Court found that the activity violated § 1: the rule of reason does not require and exhaustive factual inquiry: the reason for doing market analysis is to determine market power; however, if there is already proof of an anticompetitive effect, then market analysis is unnecessary and the boycott is illegal under the rule of reason. Because the refusal to deal “impaired the ability of the market to advance social welfare by ensuring the provision of desired goods and services to consumers at a price approximating the marginal cost of providing them,” the Court invalidated the boycott.

2. Reaffirmations of Per Se Rules: Despite the holding in BMI, which some lower courts interpreted as prescribing a rule of reason analysis in all antitrust cases, the Court continued to apply per se illegality to certain restraints on price.a. Arizona v. Maricopa County Medical Soc’y (1982): Physicians had an

agreement to set maximum fees they would charge for their services.

16

Page 17: Antitrust McChesney

Seventy percent of the medical practitioners in the County established a plan whereby they agreed not to charge patients more than a specified fee for identified services. The doctors argued that this agreement reduced prices to consumers by lowering information search costs, that the participating insurance companies were acting in the consumers’ interests, and that the medical profession met higher standards and should not be subject to the usual antitrust per se rule. Nevertheless the Court held that the fee-schedule was per se illegal under § 1. The Court invoked Socony-Vacuum to support its conclusion that “the anticompetitive potential in all price fixing agreements justifies their facial invalidation even if procompetitive justifications are offered for some. Those claims of enhanced competition are so unlikely to prove significant in any particular case that we adhere to the rule of law that is justified in its general application.” This approach is questionable; the program had genuine potential to lower consumer costs, and the use of a per se approach prevented consideration of the plan’s effect on prices or output.

b. FTC v. Superior Court Trial Lawyer’s Ass’n (1990): The Court reversed its move towards rule of reason treatment of boycotts when in held applied per se illegality to an agreement by attorneys not to represent indigent criminal defendants until the government increased the fees for such work. The Court ignored evidence regarding market power, and the fact that the boycott was designed to effect political change (an exception to the antitrust laws under the “Noerr-Pennington Doctrine” (Justice Brennan would have found the boycott legal on this ground)), and viewed the activity as a naked restrain on price and output.i. Price Fixing: The Court emphasized that this case involved not

only a boycott, but price-fixing. This may have contributed to the decision to apply per se liability.

ii. Modern Approach to Boycotts: In light of SCTLA, the Court’s modern boycott decisions suggest the following standards: (1) an agreement by direct rivals to withhold their services until the price for such services is raised is a naked restraint on output and is condemned summarily (SCTLA); (2) concerted refusals to deal that pose remotely plausible efficiency rationales are evaluated with a truncated rule of reason that begins with a preliminary assessment of the conduct’s purposes and effects (Indiana Dentists); and (3) suits challenging membership policies of efficiency-enhancing collaborations require a fuller reasonableness inquiry, including a determination of the defendant’s market power (Northwest Stationers).

c. Palmer v. BRG of Georgia, Inc. (1990): This is another territorial allocation case in which the two main suppliers of bar review courses (and hence were in direct competition) entered into an agreement under which BRG was given an exclusive license to market HBJ’s bar review materials in Georgian and use HBJ’s trade name. HBJ also agreed not to compete with BRG in Georgia. In a short per curium opinion, the Court

17

Page 18: Antitrust McChesney

cited Socony-Vacuum and Topco as establishing that market division agreements involving actual or putative competitors are illegal per se. The revenue-sharing formula, plus the immediate price increase, indicated that the agreement was “formed for the purpose and with the effect of raising” the bar view course’s prices in violation of the Sherman Act. Agreements between competitors to allocate territories to minimize competition are illegal, regardless of whether the parties split a market within which they both do business or merely reserve once market for one and another for the other.

d. Horizontal Restraints and the “Quick Look”: BMI, NCAA, and Northwest Stationers demonstrate a new approach to cases involving horizontal restraints—horizontal contracts traditionally considered as per se illegal may now be evaluated under the rule of reason. On the basis of pleadings, affidavits and other “short form” filings, a court will quickly consider whether the contract(s) at issue might not be among those “which because of the pernicious effect on competition and lack of any redeeming virtue should be conclusively presumed to be unreasonable and therefore illegal without elaborate inquiry as to the precise harm they have caused or the business excuse for their use. If the practice could conceivably have some redeeming social (procompetitive) value, it should be litigated under the rule of reason. Otherwise, on the basis of this preliminary review then the matter should be disposed of under the per se rule.

3. Lower Court (Re?)Interpretations: How did lower court interpret the shift in analysis begun in BMI and NCAA? Polk Bros. v. Forest City Enterprises (7th Cir. 1985), involved an agreement by two retailers to restrict the products that each could sell in stores that would be located within a new building which the firms had cooperated to construct. Acknowledging that most contracts allocating products and markets were unlawful per se, Judge Easterbrook nonetheless held that the rule of reason should apply. “Antitrust law is designed to ensure an appropriate blend of cooperation and competition, not to require all economic actors to compete full tilt at every moment. When cooperation contributes to productivity through integration fo efforts, the rule of reason is the norm. NCAA.” Easterbrook saw the allocations at issue in this case as ancillary to the joint venture—“A restraint is ancillary when it may contribute to the success of a cooperative venture that promises greater productivity and output.” (See Addyston Pipe.) “Only when a quick look reveals that ‘the practice facially appears to be one that would always or almost always tend to restrict competition and decrease output’ should a court cut off further inquiry.” Because the main purpose of the agreement was to avoid free riding, and market power was too small to prevent customers from going elsewhere, the agreement was not unreasonable and hence not a violation of § 1.

E. APPLICABILITY OF THE SHERMAN ACT

1. Commercial vs. Non-Commercial Activities: The question here is whether the Sherman Act applies to non-profit entities.a. United States v. Brown University (3rd Cir. 1993): The issue here was

whether an agreement between the ivy league schools to offer the same

18

Page 19: Antitrust McChesney

financial aid package to all commonly admitted students offended § 1 of the Sherman Act. The district court applied a quick-look style rule of reason analysis because the Universities were non-profit entities; but concluded that the agreement was plainly anticompetitive because it eliminate price competition for students. First, the court of appeals examined whether the Sherman Act applied to the Universities at all. Non-profit organizations are, the court found, subject to the Act ass long as the activity involves or implicates trade or commerce—i.e., the exchange of money for services. The court agreed that the rule of reason was the appropriate analysis, and that the agreement was anticompetitive on its face. But the court said the district court erred when it failed to consider the proffered procompetitive justifications for the agreement. The court expressed serious doubts as to whether the justifications did in fact apply to the agreement, and whether the restraints imposed by the agreements had were necessary to achieve its purported goals. Nevertheless, the court remanded for a full rule of reason analysis.

b. DELTA Rescue v. Humane Soc’y of the U.S., Inc. (9th Cir. 1995): The question here was whether the charities were involved in “trade or commerce,” and hence were subject to the antitrust laws. The court held that trade or commerce involves contractual type relationships, as opposed to donative relationships, which implicate property-type rights. “The solicitation of contributions by a nonprofit organization is not trade or commerce, and the Sherman Act has no application to such activity.”

2. Other Applicability Issues: Most exemptions to the antitrust law require affirmative law passed by Congress.a. Labor Unions: Because labor unions function by a contract, whereby

their members ban individual negotiations with an employer in favor of negotiation by the union, they were treated as unreasonable restraints of trade at common law. Section 6 of the Clayton Act provides labor unions with a statutory exemption from the antitrust law.

b. Insurance: Federal law not specific to insurance shall not be construed to invalidate, impair, or supercede any state regulation of the insurance industry, as long as the activity regulated by the state is part of the business of insurance and no issue of boycott, coercion, or intimidation arises. Because antitrust does not specifically relate to insurance, it is inapplicable to insurance companies.

c. Professional Sports: Obviously, sports leagues must restrict output and hence competition in order to function. Baseball has long enjoyed a judicially-created immunity from antitrust.

F. PROVING THE EXISTENCE OF CONSPIRACIES

1. The Limits of Circumstantial Evidence: Outcomes in § 1 litigation often depend on whether the plaintiff has supported its conspiracy claim with enough evidence to warrant a trial and to have its case submitted to the jury. The § 1 plaintiff bears the burden of proving an agreement. This requires a showing that the existence of an agreement is more likely than not; conclusory allegations of concerted action are prone to summary dismissal. In other words, circumstantial

19

Page 20: Antitrust McChesney

evidence of an agreement is sufficient to support such a finding, but the evidence must tend to exclude the possibility of unilateral action.a. Matsushita Elec. Industr. Co. v. Zenith Radio Corp. (1986): This was a §

1 conspiracy claim by two American electronics firms that a group of Japanese television producers had conspired to charge predatorily low prices for goods sold in the U.S., using monopoly profits from the Japanese market to subsidize the below cost pricing in the U.S. The district court granted summary judgment for the defendants on the ground that the plaintiffs had failed to put forward any evidence of an conspiracy or collusion. The court of appeals reversed, but the Supreme Court reinstated the judgment of the district court. To survive the motion for summary judgment, the plaintiff must show that there is a genuine issue of fact as to whether a conspiracy existed that caused plaintiffs to suffer a cognizable injury.i. Antitrust Injury: First, the Court says that plaintiff did not suffer

any “antitrust injury”—the conspiracy to raise prices in the U.S. market would actually benefit the American companies. Lower prices, by themselves, do not create antitrust injury.

ii. Predatory Pricing: Thus, the only claim on which the plaintiff could prevail was a showing that the low prices were intended to eliminate competition as a predicate to later raising prices to supracompetitive levels. The Court noted that predatory pricing is so speculative as to be uneconomical. Furthermore, predatory pricing as part of a combination of companies is even more unlikely.

iii. Evidentiary Standard: To survive a motion for summary judgment, a plaintiff seeking damages for a violation of § 1 must present evidence “that tends to exclude the possibility that the alleged conspirators acted independently.” Courts should not permit factfinders to infer conspiracies when such inferences are implausible, because the effect of such practices is often to deter procompetitive conduct.

iv. Balancing: In order to avoid punishing procomptetitive behavior, courts should balance that desire against the desire that illegal conspiracies be identified and punished. When the economic realities make clear that the activity at issue is unlikely to be anticompetitive, a higher quantum of evidence is needed overcome a motion for summary judgment. “If the defendants’ had no rational economic motive to conspire, and if their conduct is consistent with other, equally plausible explanations, the conduct does not give rise to an inference of conspiracy.”

v. Type I v. Type II Error: Because this case involved a competitor complaining about lower prices, the Court was reluctant to sanction the prices scheme because it was actually beneficial to consumers.

b. Monsanto Co v. Spray-Rite Serv. Corp. (1984): The Court said “the correct standard is that there must be evidence that tends to exclude the

20

Page 21: Antitrust McChesney

possibility of independent action by the parties. That, there must be direct or circumstantial evidence that reasonably tends to prove that the parties had a conscious commitment to a common scheme designed to achieve an unlawful objective.”

2. The Extent of Conspiracies: Even when the existence of some illegal activity is clearly established, the precise nature of the agreement—or agreements—and who participated in what can become a matter of vigorous debate and critical legal significance both as to criminal and civil liability.a. Single-Schemes v. Multiple-Conspiracy Characterizations: Whether the

conspiracy was a single overarching agreement, or a series of agreements involving various subsets of defendants, can have important consequences.i. United States v. Beachner Construction Co. (10th Cir. 1984): The

defendant was indicted for his part in a bid rigging conspiracy. He was acquitted, but later retried for three different acts of bid rigging. The court held that the bids in the second case were part of a single, decades-long conspiracy. Thus, to allow a second prosecution would violate the Double Jeopardy clause of the Fifth Amendment. “The pertinent inquiry is whether the record is sufficient to establish that all conspirators had a single, common and continuing objective. . . . If so, there would be but one conspiracy even though its purposes were advanced by diverse parties.”

ii. Sargent v. United States (3rd Cir. 1986): “An agreement to rig bid wherever and whenever possible is meaningless for Sherman Act purposes unless there are in the real world of the marketplace some “whens” and “wheres.”

b. Why The Number of Conspirators Matters in Civil Cases: Antitrust violations are analyzed as joint torts subject to the general tort law doctrines. Thus, the longer a defendant remains in a case involving multiple defendants without settling, the greater his share of potential liability. Ironically, this whipsaw effect is at its strongest when the merits of the liability case, as measured by the probability of liability, are at their weakest.

3. Common Purpose and Other Requirements: A conspiracy under § 1 requires proof that the conspirators had a common purpose. Also, other generally requirements of tort liability, such as causation, are necessary to a showing of antitrust liability.

G. LEGALLY AND ECONOMICALLY AMBIGUOUS PRACTICES

1. Agreements to Exchange Information: Exchanges of information between competing or related businesses often raise antitrust problems. Information exchanges have § 1 implications in essentially two settings—where data is disseminated through the assistance of a trade association, and where firms obtain or provide information by contacting competitors directly. It is important, however, to be sensitive to whether the exchange helps perfect the market or creates efficiencies, or whether it facilitates cartelization and lessens competition.

21

Page 22: Antitrust McChesney

a. Classic Cases on Information Exchange: Early cases dealing with “open competition” plans (which admittedly sought to “keep prices at reasonably stable and normal levels”) found that such plans violated the Sherman Act.i. American Column & Lumber Co. v. United States (1921): A

trade association whose members produced one-third of the nation’s hardwood lumber had adopted a plan requiring member firms to submit price lists, detailed daily sales and shipment reports, monthly production and stock reports, etc. The plan was supposed to disseminate among members accurate knowledge of production and market conditions so that each member may gauge the market intelligently instead of guessing at it. The Court held that the plan violated § 1 because it was used “to suppress competition by restricting production.” “Genuine competitors do not make . . . reports of the minutest details of their business to their rivals, as the defendants did.”

ii. Maple Flooring Mfrs. Ass’n v. United States (1925): The trade association disseminated information detailing members’ average costs and summarizing all sales, prices, and stocks—but not identifying current individual transactions; a single base point freight rate booklet (which would facilitate uniform pricing) was also distributed. The Court focused on the fact that the members never discussed future price, only past transactions. The Court basically held that competitors who meet to discuss past transactions, current prices and shipping rates, do not violate § 1.

iii. United States v. Container Corp. of America (1969): The defendants (18 separate firms) shipped 90 percent of the cardboard cartons supplied in the Southeast. They established an informal price exchange whereby suppliers gave each other, on request, price information on their most recent sales to a particular customer. Not surprisingly, once a firm had received this information, it would often quote the same price to that customer, and buyers commonly divided orders among suppliers. The Court held that the information exchange was illegal under Socony-Vacuum because “the exchange of price data tends towards price uniformity.” “Stabilizing prices as well as raising them is within the ban of § 1.” The citation to Socony-Vacuum implied that this was a per se case; however, the Court has not rigidly applied a per se rule to information exchanges. (Indeed, Justice Fortas concurred in the judgment on the understanding that the Court was not announcing a per se rule.)

iv. United States v. United States Gypsum Co. (1978): Here the government charged that gypsum board producers had engaged in per se illegal price-fixing by checking current and future prices with rivals before giving price concessions to buyers. The defendants said they had consulted with competitors to comply with the meeting competition requirements of the Robinson-

22

Page 23: Antitrust McChesney

Patman Act. The Court held that the mere exchange of price information without intent to fix prices is not criminal price-fixing per se and must be tested under the rule of reason—simple evidence of price raising or stabilizing could not lead to a presumption that the defendants intended that result. the government must show that the defendant had intent to fix prices. An effect on price, however, may well support an inference of knowledge that the agreement would effect prices.

b. Recent Information-Exchange Cases: Container and Gypsum severely limited the circumstances in which competitors can share price data directly. The applicable standard, however, is unclear; the test applied in Container was a narrower-than-usual rule of reason test, and the Gypsum decision spoke in the measured terms of the rule of reason, yet its treatment of the facts left little doubt that price exchanges were disfavored among oligopolists.i. The Five Smiths, Inc. v. NFL Players Ass’n (D. Minn. 1992):

The League charged that agents who exchanged player salary information were pushing up salaries, and were engaged in a conspiracy to fix, raise, or maintain prices in violation of § 1. The court rejected the price fixing claim because there was no link between the information exchange—it made no economic sense (see Matsushita). The court also applied the rule of reason to the information exchange, rejecting the application of a per se rule: “even assuming that some players are competitors . . . such an exchange of information among competitors is not within any of the categories of conduct that is so manifestly anticompetitive as to warrant per se condemnation.” “The Supreme Court has determined that, absent some agreement between competitors to restrain price, the exchange of price and other market information is generally benign conduct that facilitates efficient economic activity.” Furthermore, the plaintiffs failed to show any antitrust injury to competition (e.g. fewer players, fewer games, etc.).

ii. In re Petroleum Products Antitrust Litigation (9th Cir. 1988): The plaintiff alleged that oil companies were conspiring to raise prices. Plaintiffs claimed that defendants were exchanging information regarding prices through press releases and other avenues, in order to ensure an uniform price and decrease competition. The defendants had basically no justification for publicly announcing price changes other than to “facilitate either interdependent or plainly collusive price coordination.” That evidence was, therefore, sufficient to support a finding of an agreement, whether express or tacit, to raise or stabilize prices. On remand, it is therefore likely that the district court will apply a per se rule for the information exchange that was clearly intended to effect prices.

2. Oligopolistic Interaction and Facilitating Devices: Antitrust doctrine and enforcement policy have taken several paths to frustrate efforts by oligopolists

23

Page 24: Antitrust McChesney

to coordinate their behavior by means other than a direct exchange of assurances.a. Conscious Parallelism and Tacit Agreements: “Conscious parallelism”

is a process, not in itself unlawful, by which firms in a concentrated market might in effect share monopoly power, setting their prices at a profit-maximizing, supra-competitive level by recognizing their shared economic interest and their interdependence with respect to output decisions. In concentrated markets, the recognition of interdependence can lead firms to coordinate their conduct simply by observing and reacting to their competitors’ moves. This may generate parallel price movements that seem like price-fixing agreements.i. City of Tuscaloosa v. Harcros Chemicals, Inc. (N.D. Ala. 1995):

The defendants were oligopolists selling chlorine, a homogenous product, in a largely inelastic market. The plaintiffs claimed that the firms engaged in a price-fixing conspiracy, evidenced only by the fact that the market structure was such that collusion was likely. Because there was no direct evidence, plaintiffs relied on expert testimony that a price-fixing conspiracy was possible. The court held that this type of tacit agreement is not a violation of the antitrust laws. (This should be contrasted with actual agreements but of which there is only indirect evidence.) In this case there are several factors that would make price fixing easier, but no evidence of actual price fixing; thus, the court rejected the expert testimony on conscious parallelism.

ii. Plus Factors: Courts typically require plaintiffs who rely on parallel conduct to introduce additional facts, often termed “plus factors,” to justify an inference of agreement. Some plus factors recognized by courts include: whether defendants have a motive to engage in a conspiracy; whether the disputed conduct would have contradicted the defendants’ self-interest if pursued unilaterally; proof that defendants priced uniformly where price uniformity was improbable without an agreement; past antitrust violations; direct communication with competitors, and subsequent simultaneous changes in behavior; agreements to adopt common practices, such as product standardization.

b. Facilitating Devices: Facilitating devices are commercial practices or institutions that a group of conspirators may employ in order to make it easier to attain or stabilize an anticompetitive arrangement of some type. Information exchanges are a classic example of a facilitating device.i. Using Facilitating Devices to Strengthen Anticompetitive

Agreements: These practices share, along with information exchanges, the problem that they have legitimate procompetitive uses in addition to having a potential as anticompetitive facilitators.(a). Basing Point Pricing: One way to prevent members of a

cartel from chiseling on freight charges is to set a single freight rate for all shipping. The full selling price of the

24

Page 25: Antitrust McChesney

product would then be set as the product priced f.o.b. plus the freight rate from a single location (the “basing point”). Thus, the product would be charged as if shipped from Pittsburgh, even if it was actually shipped from San Francisco. In FTC v. Cement Institute (1948), the Court upheld the FTC’s finding of illegality for concerted use of basing-point pricing.

(b). Most Favored Nation Clauses: A MFNC is a provision in a multi-period commercial contract whereby the seller guarantees the buyer the right to have product delivered at either the contract price or the lowest price charged by the seller to any other customer during the term of the contract. This is another method of dissuading cartel members from chiseling.

ii. Facilitating Devices In Oligopolistic Settings: It is not permissible in most instances for rivals to form arrangement whose effect on price is indirect.(a). Catalano, Inc. v. Target Sales, Inc. (1980): Defendants

had formerly given short-term trade credit to beer purchasers. Plaintiffs alleged that the agreement between the defendants to end this practice was a per se violation of § 1. Because trade credit was an area in which the defendants had formerly competed, the Court held that the agreement to refrain from extending such credit was a form of price fixing, and was a method to raise prices. Thus, the agreement was anticompetitive on its face and was illegal per se.

(b). Non-Price Restraints: Note that certain restraints, while not explicitly on price or output, might have such a direct effect on those factors that they might trigger per se (or at least quick look) analysis.

III. MONOPOLIZATION

A. MONOPOLY POWER

1. Defining the Offenses: The Sherman Act does not condemn the mere possession of monopoly power. Because § 2 prohibits conduct which “monopolizes,” courts focus instead on the monopolist’s purpose and intent—on its positive drive to seize or exert monopoly power. In Aspen Skiing, the Court said the elements of the monopolization offense are (1) the possession of market power accompanied by (2) some kind of anticompetitive (exclusionary) conduct.a. United States v. Aluminum Co. of America (2nd Cir. 1945): The real

question in this case is to what standard is single-firm behavior held under § 2. Judge Hand suggests that § 2 is concerned with maintaining “small units,” or competitors, in the relevant market. A monopoly is basically the functional equivalent of a contract, combination, or conspiracy.

25

Page 26: Antitrust McChesney

i. Facts: When the government filed suit, Alcoa was the only domestic producer of virgin ingot; thus, its only competition was imported ingot, which gave Alcoa a 90 percent market share. Alcoa had also entered into contracts with foreign producers in which it secured covenants not to import into the United States, or to do so under restrictions, which in some cases involved the fixing of prices.

ii. Market Definition: The importance of market definition cannot be underestimated in these cases. Judge Hand defined the market narrowly to include only virgin ingot, excluding secondary aluminum because Alcoa controlled the initial production of everything that eventually ended up as secondary aluminum. He also excluded foreign competition because shipping is expense and thus Alcoa could control the amount imported simply by setting its prices. Hand says that 33 percent market share would be insufficient to establish monopoly power; 60-64 percent is doubtful; but that 90 percent is certainly sufficient to establish monopoly power.

iii. Holding: Hand concluded that it is sufficient to establish a violation of § 2 by showing that a firm having monopoly power purposefully and intentionally acquired, maintained, or exercised that power. The narrow holding was the Alcoa had violated § 2 by using its monopoly power to exclude new entrants into the market by buying up raw materials over what was necessary to meet its production requirements. The reasoning used to reach that decision, however, is more important than the holding itself.

iv. Reasoning: Hand distinguished between a firm that achieved monopoly power by “superior skill, foresight and industry,” or a firm that had monopoly power thrust upon it, from a firm that achieved a monopoly through the exclusion of competitors or other unduly coercive means. Alcoa gained its monopoly power by price fixing and erecting barriers to entry. Hand refused to consider the fact that Alcoa had only enjoyed 10 percent profits because overall profits don’t fully reflect the profits in the relevant market.

v. Significance: The holding in Alcoa established that conduct that might otherwise be lawful can violate § 2 if the actor has significant market power.

b. Economics: The “Dominant Firm” Model: A company can be a single seller of a product and yet not a “monopolist” in Sherman Act terms. Conversely, a company may be one of many sellers and have substantially less than 100 percent market share and nonetheless satisfy the market power requirements that define a “monopolist.” The Dominant Firm Model suggests that a firm may have the power to control prices even though there are many other firms competing in the relevant market. The Model presumes that the DF has the ability to produce product more cheaply that the “competitive fringe.” At a certain

26

Page 27: Antitrust McChesney

price, the fringe firms will supply the entire demand; but at a lower price, the fringe suppliers will not meet the demand and the DF can fill the residual demand. Thus, the DF can set the price as long as the fringe firms meet a certain amount of the supply and demand. There is therefore no competition with respect to this residual demand. The DF’s position may be the result of natural factors, or perhaps anticompetitive activities.

2. Sources of the Monopoly Power: Relevant Markets: Without market power, actual or probable, there is little reason to be concerned with the acts of a single firm under the antitrust laws. Thus, the definition of the relevant market is critical to any analysis under § 2. To define the product market in which the seller operates, one seeks to locate all substitutes available to buyers of the seller’s product.a. Blue Cross & Blue Shield of Wisc. v. Marshfield Clinic (7th Cir. 1995):

Plaintiffs claimed that defendants were exercising monopoly power in the HMO “market” in the counties in which the plaintiff operated. The key question in this case was the definition of the relevant product market; the answer depends on to what extent the services at issue can be substituted for—cross-elasticity of demand. Judge Posner says that HMOs by themselves are not a relevant market because the HMO is just a way for marketing doctors’ services. If HMOs raised prices, consumers would substitute PPOs or other doctors’ services, and thus HMOs compete with those entities. Thus, plaintiff only controlled 50 percent of the relevant market, “below any accepted benchmark for inferring monopoly power from market share.” Furthermore, Posner refused to infer monopoly power from plaintiff’s high prices and high profit—high profit is not, in economic terms, necessarily indicative of monopoly power, and high prices might just be a reflection of the quality of services provided.

b. American Key Corp. v. Cole National Corp. (11th Cir. 1985): The important issue in this case was the relevant geographic market. The court ruled that the geographic market consists of the area from which the sellers of the relevant product derive their customers, and the area within which purchasers of the product can practically turn for such products or services. “The relevant market is the “area of effective competition” in which competitors generally are willing to compete for the consumer potential, and not the market area of a single company.” Thus, the defendant lacked significant market power, and plaintiff was unable to maintain a § 2 claim.

3. Attempted Monopolization: In addition to prohibiting monopolization and conspiracies to monopolize, § 2 proscribes attempts to monopolize. Conduct amounts to an attempt to monopolize if there is a specific intent to monopolize and a dangerous probability that, if unchecked, such conduct will ripen into monopolization.a. Abcor Corp. v. AM International, Inc. (4th Cir. 1990): Defendant

manufactured printing equipment; plaintiffs provided maintenance and repair services for defendant’s machines in the Washington, D.C. area.

27

Page 28: Antitrust McChesney

The three elements of an attempted monopolization claim are: (1) the defendant formed a specific intent to monopolize the market; (2) the defendant engaged in anticompetitive or predatory conduct designed to further that intent; and (3) a dangerous probability of success.i. Intent to Monopolize: Attempted monopolization requires a

specific intent to destroy competition or build a monopoly. A desire to increase market share or even to drive a competitor out of business through vigorous competition on the merits is not sufficient. The court found not intent to monopolize—a desire to destroy a competitor though competition is not enough. Note the risk of Type I/Type II error in this situation. McChesney says this element is in essence just a pleading requirement—intent can be inferred from anticompetitive conduct (see Spectrum Sports).

ii. Anticompetitive Activity: Plaintiffs must also show that the defendant engaged in anticompetitive conduct—i.e., conduct designed to destroy competition. It is important to distinguish this conduct from conduct that is merely competitive. In short, the defendant must be employing unfair means to achieve his goal (the same activities that are condemned in cases of actual monopolization). Here, the court found that the defendant’s actions were not unfair or anticompetitive.

iii. Dangerous Probability of Success: A finding of dangerous probability of success is basically a function of market share/power. A dangerous probability of success can occur at market share levels well below those needed to establish actual monopolization. Any change in market share brought about by the activity at issue may also be relevant.

iv. Antitrust Injury: As always, the plaintiff must show antitrust injury, not just individual injury.

b. Spectrum Sports v. McQuillan (1993): The Court adopted the three part test for an attempt to monopolize. The Court rejected, however, the assumption that a dangerous probability of success could be inferred from proof of predatory conduct alone. The Court held that satisfying the dangerous probability element required an “inquiry into the relevant product and geographic market and the defendant’s economic power within that market.

B. EXCLUSIONARY CONDUCT

1. Exclusionary Contracts: The “bad conduct” requirement of § 2 is often very difficult to understand. It is the means of winning the competitive wars that can give rise to antitrust issues. For example, a violation may be found for exclusionary conduct that unnecessarily impairs or restricts the ability of competitors to compete. Note, however, that a defendant may be able to avoid liability by demonstrating procompetitive reasons for its behavior (see Microsoft).a. United States v. United Shoe Machinery Corp. (D. Mass. 1953): It is

unquestioned that defendant has a monopoly on shoe making machinery. The question was whether it was maintaining its monopoly through

28

Page 29: Antitrust McChesney

exclusionary conduct. The conduct at issue was a “lease-only” policy, in which defendant would only lease its machines to customers. The court found that its refusal to sell machines enhanced its monopoly position in several ways: first, it solved the problem of a secondary market in defendant’s machines; second, the “full-capacity” clause in the lease directly restricted the customer’s ability to use a competitor’s equipment; third, the price conditions made it cheaper to renew with defendant than to acquire a competitor’s machine; fourth, it provided for free service which deterred customers from switching to competitors. These restrictions led to an almost absolute barrier to entry, and was thus harmful to competition. Customers, however, were happy with the state of the market; the restrictions of the leasing policy forced defendant to give price breaks. The court’s remedy therefore is to end the leasing policy (“conduct remedy”) as opposed to breaking up the company (“structural remedy”). This case, like Alcoa, demonstrates that § 2 is less concerned with prices and quantities, and more concerned with maintaining competition.

b. Problem: The problem with the approach of United Shoe and Alcoa is that it makes otherwise acceptable conduct illegal when the firm has significant market power.

c. United States v. Microsoft Corp. (D.C. Cir. 2001): The issue in this case was Microsoft’s practices in maintaining its monopoly, not that it possessed monopoly power.i. Market Definition: The court defined the relevant market as Intel-

compatible PC operating systems. The court excluded Mac OS because consumers would not switch to Mac in response to a substantial price increase. The court also excluded “middleware” from the relevant market because middleware products could not effectively replace Windows. Having defined the market, the court found that Microsoft had a greater than 95 percent share.

ii. Anticompetitive Conduct: The bad acts that constituted the § 2 violation involved different types of transgressions. First, Microsoft misled its competitors; second, it entered into exclusionary contracts with OEMs, Apple, IAPs, and others (contracts giving Windows to OEMs for free in exchange for exclusively promoting IE; forcing competitors to market IE); third, actively trying to inhibit the develop of Java technologies.

iii. Procompetitive Justification: Even though the court found that Microsoft was engaging in exclusionary conduct, the court allowed Microsoft to offer procompetitive justifications for the conduct. Microsoft failed to do so, however.

2. Essential Facilities: This doctrine is a subset of the refusal to deal theory of antitrust liability. However, courts are very reluctant to find liability on an essential facilities theory under § 2.a. Florida Fuels, Inc. v. Krueder Oil Co. (S.D. Fla. 1989): The question was

whether oil bunkers owned by defendant were an essential facility to competing in the heavy marine fuel market in South Florida. The court

29

Page 30: Antitrust McChesney

described the requirements for liability under an essential facilities theory. The doctrine is basically an exception to the general rule that there is no duty to help one’s competitors.i. Control of Facility by Monopolist: First, the plaintiff must prove

that the defendant who controls the facility is a monopolist; in other words, the defendant must have monopoly power as evidenced by the defendant’s ability to control prices or exclude competition.

ii. Practicality of Duplicating the Facility: An essential facility need not be indispensable; but it must be economically unfeasible to recreate and must present a severe handicap on market entry. It is insufficient for plaintiff to show that access to the facility is simply more convenient, more economical, or less expensive. An inquiry into the practicality of duplicating the facility should consider economic, regulatory, and other concerns.

iii. Denial of Use: The monopolist controller of the facility must deny use of the facility to competitors. The court suggest, however, that this denial must be unreasonable.

iv. Feasibility of Sharing: The plaintiff must demonstrate that the defendant may feasibly provide access to the facility.

v. Business Justification: Aspen Skiing has been interpreted as adding a fifth element—whether the defendant advanced a reasonable business justification for denying access or conditioning access on terms that the plaintiff deems to be excessively burdensome.

vi. History of dealing: Although not an explicit factor in the essential facilities analysis, the court notes that in cases imposing a duty to deal, the Supreme Court has not required an unregulated monopolist, acting independently, to share its facility with a competitor with which it has had no prior history of dealing.

b. Early Essential Facilities Cases: In early cases, the Supreme Court treated essential facilities somewhat differently. In Associated Press v. United States (1945), the Court held that an agreement to exclude competitors from news gathered by the AP was a violation of the Sherman Act because “trade restraints of this character, aimed at the destruction of competition, tend to block the initiative which brings newcomers into a field of business and to frustrate the free enterprise system which it was the purpose of the Sherman Act to protect.” In United States v. Terminal Railroad (1912), the Court held that the owner of the only rail yard in St. Louis could not deny access to the rail yard to competing railroads who were willing to pay market rates for access. Both cases were decided under § 1.

3. Predatory Conduct: What kind of unilateral conduct, when undertaken by a firm with monopoly power, can constitute the type of activity that violates § 2?a. Predatory Pricing: This anticompetitive is based on a theory that a firm

with monopoly power could temporarily set prices below its own and its competitors’ costs, wait for its rivals to go bankrupt or leave the market,

30

Page 31: Antitrust McChesney

and then raise prices to supracompetitive levels. The graph below demonstrates how the theory works, and why it is economical suspect.

pm

pc

pp

qm qc qp

As the graph shows, it is very risky to undertake this approach to gaining market power. Furthermore, present value of money makes it more difficult to recoup losses later. Plus, because less quantity will be sold at the higher monopoly price it will be harder to recoup losses. Accordingly, like essential facilities cases, courts disfavor predatory pricing as a theory of liability.i. A.A. Poultry Farms v. Rose Acre Farms, Inc. (7th Cir. 1989):

Judge Easterbrook points out the primary problem with a predatory pricing theory of § 2 liability: it punishes firms for pricing too low; this is good for consumers, and good for competition. Thus, Type I and Type II error become a serious problem, and courts should be reluctant to discourage what might be procompetitive behavior. Easterbrook identifies factors to distinguish between aggressive competition and predatory pricing.(a). Prices & Costs: First, courts should determine whether the

defendant’s prices exceed its costs: if price exceeds cost, then it reflects beneficial aggressive competition; if the price is less than cost, then it may reflect a sacrifice in the hope of suppressing competition. Also, “monopolists set price by reference to their costs; competitors set price by reference to the market. So the statement that [defendant] does not pay attention to its own costs when setting price reveals that the firm was acting as a competitor rather than a monopolist.

(b). Intent: Second, look to the defendant’s intent. Easterbrook doesn’t like relying on this factor because it is irrelevant if the scheme fails.

(c). Possibility of Success: Third, look at the back end—if a monopoly price later is impossible, then predatory pricing is unlikely. Courts favor the third factor whenever possible. Determining the possibility of future monopoly power requires an analysis of the market structure—if the market has many competitors, if it is easy to enter, and the

31

D

MC

Anticompetitive Gains (uncertain & later)

Predatory Losses (certain & immediate)

Page 32: Antitrust McChesney

defendant’s market share is low, it is unlikely that a predatory price scheme would succeed.

ii. Brooke Group Ltd. v. Brown & Williamson Tobacco (1993): The Court emphasized the importance of “recoupment” in predatory pricing cases. If the defendant cannot recoup its losses from below cost pricing, antitrust concerns are not implicated because low prices benefit consumer welfare; and consumer welfare is the goal of antitrust. The Court sustained the dismissal of a predatory pricing claim because the plaintiff had failed to show how the defendant could recoup its investment in below-cost sales. The Court said that plaintiffs who claim predatory pricing under § 2 or under the Robinson-Patman Act must satisfy two requirements, which the Court called “not easy to establish.” First, the plaintiff must “prove that the prices complained of are below an appropriate measure of its rival’s cost. The Court did not, however, prescribe what measure of cost (average variable cost or average total cost) should be the threshold. The second requirement “is a demonstration that the defendant had a reasonable prospect, or under § 2 a dangerous probability, of recouping its investment in below-cost prices.”

b. Theories of Predatory Behavior: The Court’s increasingly skeptical view of predatory pricing allegations mirrors economists’ own belief that, for the most part, the simple sort of predatory pricing alleged in most cases makes little economic sense. Further, recoupment is impossible when there are no barriers to entry; as soon as the predator begins to price above cost, new entrants (or even the former rivals who had left the market) will compete away any attempt at monopoly pricing. Some theories, however, might allow for such predatory pricing to work. For example, if the firm anticipated predatory pricing from the beginning, it could heavily invest in fixed costs, thus lowering marginal cost and deterring entry into the market. Another theory posits that a firm might be able to raise its rivals’ costs, thus driving them out of the market and obtaining a monopoly.

4. Price Squeezes: This may arise when there are two related markets; the monopolist in the upstream market might raise prices at that level, and then lower its prices in the downstream market to drive out competition at that level.a. Town of Concord v. Boston Edison Co. (1st Cir. 1990): The narrow

question in this case was whether § 2 forbid a governmentally regulated firm with fully regulated prices (at both levels) from asking regulators to approve prices that could create a price squeeze? Judge Breyer answers the question no, based on the fact that an upstream monopolist has no economic incentive to attempt to monopolize downstream.i. Facts: The plaintiffs were towns in Massachusetts who purchased

electricity from the defendant, which was an “investor-owned utility” that operated at all three levels of the industry: (1) production of electricity; (2) transmission of electricity from generators to local distributors; and (3) distribution of electricity

32

Page 33: Antitrust McChesney

directly to consumers. Defendant’s rates were regulated by the federal government. Defendant petitioned the government to raise the rates it charged to local distributors; thus, while the rates local distributors paid for electricity rose, the rates defendant charged to consumers stayed the same. This created the so-called “price squeeze.”

ii. Reasoning: Breyer begins by noting that the purpose of antitrust is lower prices, better products, and more efficient production methods. Furthermore, antitrust rules must be simple enough for lawyers to explain to clients. Also, antitrust is not concerned with prices per se, but with processes that result in certain prices. The first problem with the price squeeze theory is that it is basically complaining about lower prices, which is problematic ab initio. Furthermore, and most importantly, Beryer says the price squeeze in this case is economically counterintuitive: an upstream monopolist has no incentive to monopolize downstream because it can reap all the monopoly profits it needs at the first level; in other words, a production level monopolist doesn’t care what happens downstream. This reality is only untrue if there is also a monopolist downstream; but that would have the effect of forcing the upstream monopolist to lower its prices and increase its output in the downstream market, which is good from an antitrust point of view; and it would allow the upstream monopolist to “squeeze” the downstream monopolist into lower prices, also good.

iii. Holding: Breyer therefore holds that any anticompetitive risks associated with a price squeeze are outweighed by the possible economic benefits; and that, in any event, there is not much economic incentive to monopolize the downstream market. Because the risks of anticompetitive effects are even more remote in a regulated industry (removes the potential barrier to entry), price squeezes in a regulated industry do not violate § 2.

33

MCp

MCp + MCw

Dc

Dw

MRcMRw

QcQs

MCw = $1

Ps

Pc

Pw

Pc – Pw = $1

TWO-LEVEL MONOPOLYMCp – Marginal cost of production

MCp + MCw – Marginal cost of wholesaling

Dc – Consumer demand

MRc – Marginal revenue for production and wholesaling

SINGLE MONOPOLYMRw – Marginal revenue for production only

Dw – Consumer demand

Ps – Successive monopoly price

Graphic Illustration Of Breyer’s Decision

Page 34: Antitrust McChesney

C. NONCOLLUSIVE, NONMONOPOLIZING, NONCOMPETITIVE CONDUCT

1. E.I. DuPont De Nemours & Co. v. FTC (“Ethyl Case”) (2nd Cir. 1984): This case was brought under § 5 of the FTC Act, which challenged the simultaneous adoption by the two leading firms in the ethyl industry of the following practices: basing-point pricing, advance notice to competitors of price increases, and use of most favored nation clauses. The FTC concluded that, although the adoption of these practices was non-collusive, they collectively had the effect of substantially lessening competition by facilitating price parallelism at noncompetitive levels higher than might have otherwise existed. Thus, the claim is basically one of tacit collusion; which, however, is a problem because there are no “plus factors” that make it a violation of the Sherman Act. The Second Circuit held that practices were not of the type that the antitrust laws are concerned with; and, because the FTC failed to show the practical anticompetitive effects of the practices, there was no harm to competition. The court noted that may of these practices were adopted decades before the antitrust suit was brought, and that there was evidence of substantial non-price competition between the firms. The court also noted that § 5 cannot be violated by non-collusive, non-predatory, and independent conduct of a non-artificial nature. Liability under § 5 for “unfair practices” requires, at a minimum, a showing of “oppressiveness” such as (1) evidence of anticompetitive intent or purpose on the part of the producer charged, or (2) the absence of an independent legitimate business reason for its conduct. McChesney say, “No contract, no agreement, no antitrust injury, no problem.”

2. Standards of § 5 Violation: While specific standards are elusive, the FTC has described various factors it will consider in determining whether a practice not covered by the letter or spirit of other laws in nonetheless “unfair”: whether the practice offends public policy; whether it is immoral, unethical, oppressive, or unscrupulous; and whether it causes substantial injury to consumers.

IV. VERTICAL RESTRAINTS

A. INTRODUCTION

1. In General: Vertical relationships are between firms at different levels of production or distribution. Antitrust law creates an incentive for companies to vertically integrate without independent contracts because it will eliminate the potential for § 1 liability. But this is not the most economical way to do business. For firms that do contract along the vertical supply chain, in order to extract the full potential gain they frequently condition their contractual relationships in ways designed to regulate the conduct of the other party. Such restraints can have both pro- and anticompetitive results.

2. The Economies of Contracting Out—Potential Gains and Problems: It is clear that a firm producing a product can reap larger revenues if it contracts-out the process of marketing the final product to consumers, rather than assume the additional costs of marketing the product itself.a. Potential Gains: The graph below demonstrates how contracting out can

increase profit. If the firm produces and markets the product itself, it produces at Q0, and makes profit A; if it contracts with a distributor who

34

Page 35: Antitrust McChesney

can market the product at a lower cost, it would allow the producer to sell additional units up to Q1; and to make additional profits B + C.

Note that the economic effect is the same as above when the agent/distributor doesn’t lower costs, but instead raises the price and thus raises marginal revenue.

b. Potential Problems: Independent distributors have an incentive to free-ride on the efforts of other distributors (sell a competing product at a lower price, using the producers efforts at product demonstrations, etc., to educate the consumer), or otherwise to pursue their own interests at the expense of the manufacturer. Vertical restrains certainly protect the manufacturer from its distributors’ deviations from behavior maximizing the manufacturer’s profits. But vertical restrictions also protect faithful distributors from the free-riding of other distributors.

B. COMPETITIVE THREATS VS. COMPETITIVE OPPORTUNITIES

1. Anticompetitive Concerns: Although firms might contract with distributors to stabilize demand or to concentrate sales and advertising efforts within a certain territory, such restraints may merely mask efforts by manufacturers or dealers to fix prices horizontally.a. Dr. Miles Medical Co. v. John D. Park & Sons (1911): A manufacturer

of a patented medicine sued a wholesaler on the ground that the latter obtained the plaintiff’s medicine at cut prices by inducing others to breach their price agreements with plaintiff. The Court held that a manufacturer who sells goods to a wholesaler may not restrict their resale by constraining the buyer’s pricing decision. This is known as “Resale Price Maintenance,” and continues (with some exceptions) to be per se illegal under § 1. The plaintiff was basically setting a minimum price at which its product could be resold—tantamount, in the Court’s opinion, to price fixing. This decision was based on the somewhat artificial notion that once the wholesaler purchases the goods, the contract restricting price would be a restraint on alienation, illegal at common law. In dissent, Holmes says the holding relies too much on formal distinctions—the Court ignores the economic purpose of the contract and its effect on consumer welfare; and, therefore, the Court should be reluctant to apply per se liability.

b. United States v. General Electric Co. (1926): The Court maintained the formal property-based distinction in Dr. Miles in holding that if the

35

Marginal Cost

Marginal Revenue

A

B

C

Q0 Q1

Profit Effect of Agent/Distributor Whose Marketing Yields The Same Revenues at Lower Costs

P

Q

Page 36: Antitrust McChesney

manufacturer’s agent/distributor never takes title to the goods (if the manufacturer continues to bear the risks consequent to ownership), the antitrust laws allow it to dictate the terms of sale—including retail prices. This continued reliance on formal distinctions focused on the method of the producer’s control and not its effect on competition or consumer welfare, the supposed goal of antitrust.

c. Simpson v. Union Oil Co. (1964): The Court rejected the holding in GE, and ruled that Union Oil violated the Sherman Act by fixing retail prices for its service stations-consignees. The Court reasoned that Union Oil was suing the vertical price restraint to “coerce” nominal agents.

d. Vertical Contracts vs. Unilateral Action: The formal distinctions of Dr. Miles and its progeny make the definition of “agreement” very important in vertical restraint cases. As GE implied, if the defendant is acting unilaterally, it cannot run afoul of § 1.i. United States v. Colgate & Co. (1919): The Court held that a

manufacturer’s advance announcement that it would not sell to price cutters did not violate the Sherman Act. “In the absence of any purpose to create or maintain a monopoly, the Act does not restrict the long recognized right of trader or manufacturer engaged in an entirely private business, freely to exercise his own independent discretion as to parties with whom he will deal; and, of course, he may announce in advance the circumstances under which he will refuse to sell.” Thus, the Colgate doctrine basically allowed manufacturers to “suggest” a retail price, and then refuse to deal with wholesalers who did not sell at the suggested price.

ii. United States v. Parke, Davis & Co. (1960): The defendant drug manufacturer sought to obtain compliance with its “suggested retail price” plan by bargaining and mediating with retailers who refused to abide by the prices suggested by defendant. In essence, the goal and primary effort was indistinguishable from that of Colgate; the only difference was the Parke, Davis did not have to terminate the dealer. Nevertheless, the Court held that the “agreement” violated the Sherman Act.”

e. Albrecht v. Herald Co. (1968) (Note: the Court expressly overruled this case in State Oil v. Khan): The defendant newspaper refused to sell to the plaintiff-distributor when the latter resold the papers to customers at more than the suggested retail price. The Court held that this maximum price fixing arrangement was per se illegal under § 1. “Agreements to fix maximum prices no less than those to fix minimum prices, cripple the freedom of traders and thereby restrain their ability to sell in accordance with their own judgment.” Justice Harlan dissented on the ground that this formal approach ignored the economic realities of the situation.i. Practical Effect: The practical economic effect of the Court’s

decision in Albrecht was for manufacturers to integrate downstream. This is bad because it eliminates independent distributors, and is economically wasteful.

36

Page 37: Antitrust McChesney

ii. Paschall v. Kansa City Star (8th Cir. 1984): The newspaper vertically integrated and eliminated the downstream distribution market. The court held this was not per se illegal, but subject to a rule of reason analysis.

f. State Oil Co. v. Khan (1997): The Court overruled Albrecht in holding that an agreement between a manufacturer and distributor under which the distributor had to sell at the manufacturer’s suggested retail price or rebate the amount of the overcharge to the manufacturer (maximum price fixing), was subject only to rule of reason analysis. “Our analysis is guided by our general view that the primary purpose of the antitrust laws is to protect interbrand competition. ‘Low prices,’ we have explained, ‘benefit consumers regardless of how those prices are set, and so long as they are above predatory levels, they do not threaten competition.’ So informed, we find it difficult to maintain that vertically-imposed maximum prices could harm consumer or competition to the extent necessary to justify their per se invalidation.”

g. State of the Law: Under § 1, vertical maximum price fixing is judged under the rule of reason (Khan); and vertical minimum price fixing is per se illegal (Dr. Miles). The same conduct is always per se illegal if the agreement is horizontal. This illustrates the strange treatment given to vertical agreements.

2. Accommodating Efficiencies—Relaxed Rule for Non-Price Restraints: Once resale price maintenance was foreclosed, firms began to look to other contractual means for attaining similar or additional distributional advantages. Initially, the Court ruled that even non-price vertical restraints were per se illegal. United States v. Arnold, Schwinn & Co. (1967). Later cases, however, demonstrated a more enlightened approach to vertical economic relationships.a. Continental T.V. v. GTE Sylvania, Inc. (1977): Sylvania had a negligible

market share in the national television market. In an effort to increase sales, it abandoned its old distribution methods in favor of selling to franchised retailers. The retailers, however, were restricted to certain geographic areas, and could only operate from approved locations. The strategy worked, and Sylvania’s market share increased. Dissatisfied with its sales in San Francisco, Sylvania appointed an additional dealer to service the area. The existing San Francisco dealer objected, and moved its store to Sacramento. This move violated the location clause in the franchise agreement, and Sylvania terminated the franchise. The franchisee sued, claiming the location clause was a vertical restriction that violated § 1 of the Sherman Act.i. Reasoning: The Court recognized that under Schwinn, the

territorial restrict at issue was per se illegal because the franchisee actually acquired title to the t.v. sets before it sold them to consumers—Schwinn made agency contracts subject to the rule of reason, and resale contracts subject to the per se rule. The Court recognized, however, that this approach was out of line with economic realities: “Vertical restrictions promote interbrand competition by allowing the manufacturer to achieve

37

Page 38: Antitrust McChesney

certain efficiencies in the distribution of his products. . . . Such restrictions, in varying forms, are widely used in our free market economy. There is substantial scholarly and judicial authority supporting their economic utility. . . . Accordingly, we conclude that the per se rule in Schwinn must be overruled. In so holding we do not foreclose the possibility that particular applications of vertical restrictions might justify per se prohibition. But we do make clear that departure from the rule of reason standard must be based upon demonstrable economic effect rather than formalistic line drawing.”

ii. Functional Antitrust: Justice Powell’s decision demonstrates a rejection of the formal distinctions that were embodied in Dr. Miles and its progeny in favor of a more “functional” approach to antitrust analysis. Thus (at least in the world of vertical restraints), rule of reason is the norm, and per se liability is the exception. This represents a dramatic shift in the Court’s approach to antitrust: formal distinctions should be replaced by a pragmatic examination of the economics of the activity and its effect on competition.

b. St. Martin v. KFC Corp. (W.D. Ky. 1996): This case involved a “dual distributorship” structure, in which the manufacturer has both independent franchised distributors, and also operates as a distributor itself. KFC has “company towns,” in which it acted as the only distributor, and refused to allow the plaintiff to open a franchise in Las Vegas, a company town. Plaintiffs alleged a violation of § 1in that KFC excluded franchises from company towns, and prohibited franchisees from acquiring other fast-food franchises other than KFC outlets. The court said that his was a vertical, non-price restraint, and therefore subject to the rule of reason. The “company store” clause is only a restriction on intrabrand competition, and is therefore subject to rule of reason analysis under Sylvania. McChesny says, however, that the restriction on obtaining other fast-food franchises is a horizontal restriction—it is possible for a contract to have both vertical and horizontal components. The court also cites Copperweld for the proposition that KFC’s actions were unilateral; this reliance, however, is misplaced because the franchisees are not part of the same corporation, but independent entities with only a contractual relationship with KFC.

C. APPLYING THE RULES

1. Price Agreements: Plaintiffs in the vertical world are always trying to get to the Dr. Miles rule; to have the situation characterized as retail price maintenance. But this approach is in tension with the ruling Sylvania and the notion of “functional antitrust.”a. Monsanto v. Spray-Rite Service Corp. (1984): The defendant

manufacturer terminated a price-cutting dealer after receiving complaints from nondiscounting dealers. The dismissed dealer alleged that defendant was engaging in retail price maintenance as evidenced by the fact that it had been dismissed for failing to meet the fixed price. The

38

Page 39: Antitrust McChesney

Court held, like in Matsushita, that the evidence presented to support a finding of an agreement must tend to exclude the possibility of unilateral action. “Permitting an agreement to be inferred merely from the existence of complaints, or even from the fact that termination came about ‘in response to’ complaints, could deter or penalize perfectly legitimate conduct.” The Court’s skeptical approach towards circumstantial evidence of retail price maintenance agreements makes it harder for vertical plaintiffs to prove those agreements and thus obtain per se analysis.

b. Business Electronics v. Sharp Electronics (1988): Like Monsanto, this case involved the termination of a price-cutting distributor. Defendant published a list of suggested retail prices, but its dealership agreements did not obligate the dealers to observe those prices. One dealer complained the defendant that the plaintiff was selling the product below the suggested prices. In essence, the plaintiff was free riding of the other dealers efforts to educate consumers and provide technical support. Defendant then terminated plaintiff in response to the complaints. Justice Scalia noted that the approach to cases such as this must be guided by premise of Sylvania: “that there is a presumption in favor of a rule of reason standard; that departure from that standard must be justified by demonstrable economic effect, such as the facilitation of cartelizing, rather than formalistic distinctions; that interbrand competition is the primary concern of the antitrust laws; and that rules in this area should be formulated with a view towards protecting the doctrine of Sylvania.” The Court held, therefore, that a vertical restraint is not subject to the per se rule unless it specifically relates to prices—a restraint that merely affects price is subject to rule of reason analysis. (Note that this approach itself is contrary to Sylvania because it draws a bright line between contracts specifically on price and those that simply affect price.)

c. Sportmart, Inc. v. No Fear, Inc. (N.D. Ill. 1996): This case demonstrates the fine line between unilateral price fixing and refusals to deal (Colgate Doctrine), and resale price maintenance. This case looked like Monsanto—terminating a price cutting distributor. But the plaintiff was able to offer evidence that defendant “had agreements with other retailers to sell their goods at higher prices.” Thus, the court held that there was a jury question as to whether defendant was engaged in resale price maintenance.

2. Other Restraints: The range and variety of non-price vertical restraints is nearly infinite.a. O.S.C. Corp. v. Apple Computer, Inc. (C.D. Cal. 1985): Defendant

sought to prohibit its distributors from selling product through mail orders; those sales were harming Apple’s business because it was losing money it spend on demonstrations and service (mail-order distributors were free riding on the efforts of other distributors). The per se rule doesn’t apply because the restraint doesn’t apply to price; and rule of reason analysis leads to a finding of no liability because plaintiff failed to

39

Page 40: Antitrust McChesney

show any adverse effects to competition. The court also relies on market power in its rule of reason analysis—without significant market power, there can be no anticompetitive effect, and the rule of reason analysis can end there.

b. Murrow Furniture v. Thomasville Furniture Indus. (4th Cir. 1989): This was a non-price, territorial restriction, and thus subject to rule of reason analysis under Sylvania. The court focuses on the fact that defendant lacked significant interbrand market power in holding that the restraint did not violate the antitrust laws.

3. Rule of Reason: In the area of vertical restraints, courts applying the rule of reason usually conduct the following inquiry. First, courts will look to whether the defendant has significant market power to warrant further inquiry; if not, the analysis ends there. Second, if the defendant has market power, courts then look to whether the restraint has a valid procompetitive justification. If the defendant offers such a justification, the burden shifts back to the plaintiff to show that the asserted rationales are outweighed by other evidence of harmful anticompetitive effects.

V. TYING AND EXCLUSIVE DEALING

A. BACKGROUND—POLITICS OF THE CLAYTON ACT

1. Application of the Clayton Act: Tying and exclusive dealing involve contracts allegedly in restraint of trade, which, moreover, may also be a means of monopolization. These practices, however, are not governed by the Sherman Act; rather, § 3 of the Clayton Act was passed specifically to addressed these practices.

2. Language of the Act: Section three’s language doesn’t specifically mention tying and exclusive dealing; it was, however, expressly designed to criminalize such behavior. The Act, notably, only prohibits tying and exclusive dealing when their effects “may be to substantially lessen competition or tend to create a monopoly.” Thus, the statute seems to mandate a rule of reason analysis.

B. TIE-IN SALES

1. Traditional Cases: Under a tying arrangement, the seller of a product conditions the sale of one product (the tying product) upon the buyer’s agreement to purchase a second product (the tied product). The concern is that a monopolist in the tying product market may use that leverage to garner sales in the tied product market, thereby foreclosing competition in that market. Judicial approach to these cases has focused on both on the tying arrangement itself as well as on the defendant’s market power and the affect of the arrangement on the market.a. International Salt Co. v. United States (1947): Defendant, the largest

producer of salt for commercial use, tied the lease of its salt dissolving and processing machines to the lesee’s purchase from it of all the salt used in operating the machines. The Court used a “quasi” per se rule of liability to hold that arrangement violated § 3. The Court focused on defendant’s market power and on the substantial dollar volume of business in the tied product as proving anticompetitive effect. Once

40

Page 41: Antitrust McChesney

these minimum threshold elements were shown, the Court held that the violation was proven. The Court rejected the legitimate business reasons that defendant offered to justify the arrangement—it seemed to suggest that there were less restrictive alternatives than tying to achieve the business goals defendant meant to attain through the tying arrangement.

b. Northern Pacific Railway Co. v. United States (1958): The Court termed the rule laid down in International Salt as a per se rule against tying. It also held that tying was illegal under the Sherman Act, and the same standard of illegality applied.

c. Siegel v. Chicken Delight, Inc. (9th Cir. 1971): Defendant gave away its franchise license (tying product) for free, but required the franchisee to purchase cooking equipment, food items, and packaging (tied products) from defendant. The court reasoned that there are three requirements for a finding of liability for tying: (1) that the tying and tied products are two distinct products; (2) that the defendant has sufficient market power in the tying product market (i.e., “economic power”) to restrain trade in the tied product market; and (3) that the arrangement effects a “not insubstantial” amount of commerce. (Economic power is different than market power.) The court said that sufficient market power can be presumed when the tying product is patented or copyrighted; and that the power to impose the tie-in may establish as a matter of law the existence of market power sufficient for a violation. The court also rejected the legitimate business justification for the arrangement.

d. Mozart Co. v. Mercedes-Benz of N. America, Inc. (9th Cir. 1987): The court recognized that, with respect to franchise arrangements, tying may be the most efficient and effective method of ensuring quality.

2. Recent Tying Cases: The “partial” per se test developed by the court in International Salt and Northern Pacific Railway seemed unduly rigid in practice (for example, the result in Chicken Delight). In more recent years, the Court has backed off somewhat from this formal approach.a. Jefferson Parish Hospital Dist. No. 2 v. Hyde (1984): Defendant hospital

required patients wanting operations to use the hospital’s anesthesiologists. The hospital claimed that this bundling of services reduced costs and improved the quality of care; an excluded anesthesiologist complained that the tie prevented him from obtaining privileges of practice. The Court unanimously held that this arrangement did not violate the antitrust law; the Justices disagreed, however, on why the tie was acceptable.i. Majority: Justice Stevens says for the majority that “it is far too

late in the history our antitrust jurisprudence to question the proposition that certain tying arrangements pose an unacceptable risk of stifling competition and therefore are unreasonable per se.” “It is clear, however, that every refusal to sell two products separately cannot be said to restrain competition.” “The essential characteristic of an invalid tying arrangement lies in the seller’s exploitation of its control over the tying product to force the buyer into the purchase of a tied product that the buyer either did

41

Page 42: Antitrust McChesney

not want at all, or might have preferred to purchase elsewhere on different terms. When such ‘forcing’ is present, competition on the merits in the market for the tied item is restrained and the Sherman Act is violated. Accordingly, we have condemned tying arrangements when the seller has some special ability—usually called “market power”—to force a purchaser to do something that he would not do in a competitive market. When forcing occurs, our cases have found the tying arrangement unlawful.” Justice Stevens does not apply the per se rule in this case, however, because defendant lacked market power in the tying product. Under the rule of reason, there is no anticompetitive harm because “there is no evidence that the price, the quality, or the supply or demand for either the ‘tying product’ or the ‘tied product’ involved in this case has been adversely affected by the” tying arrangement.

ii. Concurring Opinion: Justice O’Connor concluded that surgery and anesthesia were not separate products for tying purposes. More importantly, Justice O’Connor would drop the “per se” label for the inquiry under tying cases, and refocus the inquiry on the adverse economic effect, and the potential economic benefits, that the tie may have. “The ultimate decision whether a tie-in is illegal under the antitrust laws should depend upon the demonstrated economic effects of the challenged agreement.” In O’Connor’s opinion, tying is only illegal if: (1) the seller has power in the tying market; (2) there is a substantial threat that the seller will acquire power in the tied-product market; (3) there is a coherent economic basis for treated the two products as distinct; and (4) the economic effect of the tie is anticompetitive.

iii. Shift in Analysis: Even under the majority’s “per se” test, this case demonstrates a shift from an analysis focused on supply-side competition to a focus on consumer/demand-side effects.

b. Town Sound & Custom Tops, Inc. v. Chrysler Motors (3rd Cir. 1992): This case demonstrates the confusion caused by the Jefferson Parish decision. Plaintiff alleged that defendant was tying sales of new cars to sales of radios to go in the new car. The court treated the per se test as totally separate from the rule of reason analysis; but this approach illustrates how the two tests overlap. First, the court says that for the per se rule to apply, defendant must have significant market power in the tying market. Plaintiffs wanted to define the market as Chrysler cars; but this makes no sense because Chrysler cars are not so wholly distinct from other cars that consumers don’t consider other cars as substitutes for Chrysler cars. Thus, the relevant tying product market is Chrysler cars and cars that are substitutes for Chrysler cars. Because defendant lacked significant power in that market, the per se rule did not apply. The court read Jefferson Parish to say that under the rule of reason a tie may be illegal if it serves as an unreasonable restraint on competition in the tied product market. The court then analyzes this issue as basically

42

Page 43: Antitrust McChesney

an question of antitrust injury—plaintiff must state a theory of injury to competition that doesn’t require market power. Plaintiffs do show injury, but not as a result of injury to competition; the injury was caused by consumers choosing not to buy its product.

c. Eastman Kodak Co. v. Image Technical Servs., Inc. (1992): The Court further confused the appropriate analysis for tying arrangements when it suggested that a seller without market power in the market for the sale of original equipment can possess market power in the aftermarket of parts and services for that product. The tying product was replacement parts for the copy machines; the tied product was service contracts for repairs to the machines. The Court said there was a triable issue of fact as to whether the service and the parts constituted different products for tying purposes. The dissent said that lack of market power in the primary market is inconsistent with market power in the derivative aftermarket.

d. United States v. Microsoft Corp. (2001): The court refused to apply a per se rule to the tying claims against Microsoft. Notably, the court seems to ignore the forcing analysis in Jefferson Parish, and goes straight into a Sylvania-type economic analysis. The court concludes that because the technology is so new, it is impossible to determine whether the tie has harmed competition. The court instructed that on remand the government would have to prove several things. First, it must show that Microsoft’s conduct unreasonably restrained competition, which requires an inquiry into the actual effect of the conduct on competition in the tied market. This would in turn require a careful definition of the tied good market and a showing of barriers to entry other than the tying arrangement itself.

C. EXCLUSIVE DEALING

1. Traditional Approach: Another way to secure vertical integration through contract is exclusive dealing. Instead of relying on economic power in the tying product market to obtain sales of a second product, the manufacturer offers a sales contract conditioned on the buyer’s agreement not to deal in the goods of a competitor; the buyer is “tied” to a particular manufacturer. This type of arrangement may, however, have substantial procompetitive advantages; it may stabilize demand, prices, and reliability of supply, and can avoid the problem of interbrand free riding. In Standard Oil Co. v. United States (“Standard Stations”) (1949), defendant supplied gasoline to independent service stations through requirement contracts, as did all other gasoline suppliers. Despite the fact that the contracts in question only affected 16 percent of the market, the Court held that the contracts violated § 3 of the Clayton Act. The Court seemed to apply a per se rule—it said a review of the economic would be too complicated, and reasoned that in any event § 3 only required a showing that the activity “may” be to lessen competition. But the Court’s exhaustive examination of the economics of these agreements suggested something more than a simply per se rule.

2 Recent Approach: Lower courts have tended not to follow the rigid rule laid down in the Standard Stations case. Modern cases focus on three principle criteria in evaluating the reasonableness of exclusive dealing arrangements: (1)

43

Page 44: Antitrust McChesney

the extent of market foreclosure (up to 30 percent is presumably acceptable after Jefferson Parish); (2) the duration of the exclusive arrangement (short terms and short notice for termination are frequently presumed to be reasonable); and (3) entry barriers (easy entry will usually lead to a finding of reasonableness).a. Tampa Electric Co. v. Nashville Coal Co. (1961): A coal suppler argued

that its agreement to fill an electric utility’s “total requirements” for coal should not be enforced because it violated § 3. The Court upheld the contract because it found that it did not affect a substantial enough amount of the coal market to raise antitrust concerns. The Court seemed to reject the “substantial amount of commerce” analysis in Standard Stations which looked only to the actual amount of money involved. This decision seemed to move the analysis in exclusive dealing cases more towards a rule of reason.

b. Roland Machinery Co. v. Dresser Indus., Inc. (7th Cir. 1985): Defendant had a dealership agreement with plaintiff that provided for termination by either party on 90 days notice. Plaintiff also signed an agreement with a rival manufacturer. After discovering this second agreement, defendant gave notice that it would exercise its right to terminate the contract. Plaintiff’s theory was based on an implied exclusive dealing arrangement. Judge Posner begins by observing that the law of exclusive dealing is in disarray. First, Posner says defendant’s actions are analogous to behavior approved under the Colgate doctrine: there was no evidence of meeting of the minds on the exclusive dealing arrangement, and defendant’s announcement that it will not deal with distributors dealing in a rival’s product is a unilateral decision free from antitrust concern. Furthermore, Posner says that “although the Supreme Court has not decided an exclusive-dealing case in many years, it now appears most unlikely that such agreements . . . will be judged by the strict test of Standard Stations. They will be judged under the rule of reason, and thus condemned only if found to restrain trade unreasonably.” Posner says that under the rule of reason, the plaintiff must show that the exclusive dealing arrangement is likely to keep at least one significant competitor from the market, and the probable effect will be to raise prices or otherwise injure competition. In other words, “foreclosure” is no longer the gravamen of exclusive dealing—it’s really about prices and quantities.

VI. MERGERS AND ACQUISITIONS

A. CLASSIC MERGER CASES

1. Introduction: Antitrust is only interested in horizontal mergers. Mergers basically serve as an alternative to price fixing. Section 7 of the Clayton Act prohibits mergers that may “tend to substantially lessen competition or tend to create a monopoly.” The “Merger Guidelines” promulgated by the DOJ have had a significant influence on which mergers the government will challenge, as well as on the approach courts take in analyzing mergers.

44

Page 45: Antitrust McChesney

2. United States v. Von’s Grocery Co. (1966): Defendant, a chain grocery store in Los Angeles, acquired another chain grocery store; together, the two firms accounted for 7.5 percent of the market. This was also set against a trend in the market which saw the gradual elimination of smaller, independent stores and the rise of chain stores. But, at the time of the merger, there were still over 3,000 single-store operations in the L.A. market. The Court holds that these facts taken together were enough to render the merger illegal under § 7. This amounted to basically a per se rule against mergers. The Court’s decision was premised on the belief that § 7 was designed to protect “small dealers and worthy men,” which accounts for the statute’s directive that anticompetitive mergers should be halted in their “incipiency.” In dissent, Justice Stewart noted that mergers can have significant procompetitive effects, and thus a per se rule is inappropriate. Furthermore, the goal of antitrust is to protect competition, and not competitors.

3. Brown Shoe Co. v. United States (1962): The Court’s first major merger decision, and perhaps still the benchmark for mergers in unconcentrated markets. The Court held that a functional approach should guide courts in examining mergers; simply looking at market share was insufficient—the context of the industry must significantly factor into the analysis. Because the goal of merger laws was to protect markets from becoming too concentrated, the Court said that mergers must be stopped before they occur—“curbed in their incipiency.”

4. Untied States v. Philadelphia National Bank (1963): The Court blocked a merger that would create the largest bank in Philadelphia. It held that mergers resulting in a high market share were so likely to substantially lessen competition, that a detailed analysis was unnecessary. “a merger which produces a firm controlling an undue percentage share of the relevant market, and results in a significant increase in the concentration of firms in that market is so inherently likely to lessen competition that it must be enjoined in the absence of evidence showing that the merger is not likely to have such anticompetitive effect.”

5. FTC v. Procter & Gamble Co. (1967): P&G sought to acquire Clorox, the largest seller of household bleach; P&G did not did not itself manufacture or sell bleach. The Court ruled that the merger violated § 7 for two reasons: first, that the merger would give Clorox a decisive advantage in the bleach market, allowing it to raise barriers to entry; and second, that P&G was the only potential competitor for entry into the bleach market. The Court rejected the argument that the merger would bring costs down (and lower prices for consumers) for Clorox.a. The Potential Competition Doctrine: In Procter & Gamble, the Court

stated that “it is clear that the existence of Procter at the edge of the industry exerted considerable influence on the market. First, the market behavior of the industry was influenced by each firm’s predictions of the market behavior of its competitors, actual and potential”; second Procter had no barrier to entry; third; there were few potential entrants, so elimination of Procter as one would be significant; and fourth Procter most likely entrant. Thus, the Court based its decision not on the fact that the merger would diminish competition, but that competition might

45

Page 46: Antitrust McChesney

be increased if the merger was enjoined. In United States v. Falstaff Brewing Co. (1973), the Court expanded the potential competition doctrine to include firms outside the market that, although not actually considering entry, might be perceived by firms in the market as being a possible entrant, thereby affecting prices in the market. This theory has not been applied, however.

b. Market Efficiencies: Merger (and antitrust) law doesn’t adequately address the possibility of productive efficiency, and how that might outweigh the loss to consumers. As the graph below demonstrates, it is possible that a merger might result in certain efficiencies that will bring costs down, thereby lowering prices, even though the firm’s profits would increase. The Court in Procter & Gamble rejected this as a possible defense to in merger cases because the “efficiency” might act as a barrier to entry.

6. United States v. General Dynamics Corp. (1974): Relying on the imperative in Brown Shoe that “statistics concerning market share and concentration, while of great significance, are not conclusive indicators of anticompetitive effect,” the Court upheld the merger of two leading coal producers. This represented a major departure from the per se rule of Von’s and Procter & Gamble. The Court also considered the efficiencies that would result from the merger.

B. PUBLIC AGENCY ENFORCEMENT POLICIES

1. Merger Guidelines: The DOJ Merger Guidelines state that the “unifying theme” of federal merger enforcement is that “mergers should not be permitted to create or enhance market power or to facilitate its exercise.” “While challenging competitively harmful mergers, the Agency seeks to avoid unnecessary interference with the larger universe of mergers that are either competitively beneficial or neutral.” There are five steps for identifying mergers that might have anticompetitive effects: (1) does the merger cause a significant increase in concentration and produce a concentrated market?; (2) does the merger appear likely to cause adverse competitive effects?; (3) would entry sufficient to frustrate anticompetitive conduct be timely and likely to occur?; (4) will the merger generate efficiencies that the parties could not achieve by other means?; and (5) is either party likely to fail, and will its assets leave the market if the merger does not occur? The DOJ takes the following steps to answer these questions.

46

D

P

MC

MC’ (result of merger)

Deadweight loss

Income shift

Productive Efficiency

(Market Efficiencies)

Page 47: Antitrust McChesney

a. Market Definition: To assess the merger’s impact on concentration, the Guidelines use market power measurement methods. Thus, the first step is to define the relevant product and geographic markets. The focus is on cross-elasticity of demand. This is by far the most important step in the merger analysis.

b. Market Participants: Current producers and sellers, and also firms that participate through a supply response (uncommitted suppliers) are considered. The possibility of entry is also important.

c. Herfindahl-Hirschman Index: Once the market is defined, and the participants are identified, the HHI is calculated. The HHI is calculated by summing the squares of the market shares of each firm in the market. For example, if there are three firms in the market, with 10%, 40%, and 50% market shares respectively, the HHI would equal 102 + 402 + 502, or 4200. This indicates how concentrated the market is pre-merger.

d. Application of HHI: The question here is how will the merger affect market concentration. The HHI is recalculated using the post-merger market shares. If the pre-merger HHI is 1800 or above and it is increased by more than 50, the government will challenge the merger; if, the pre-merger HHI is between 1000 and 1800, and is increased by more than 100, the government will challenge the merger; if the HHI is below 1000, the government will not challenge. In the above example, if the 10% and 40% firms merged, the new HHI would be 502 + 502, or 5000, an increase of 800; thus, the merger would be challenged.

e. Efficiencies: The Guidelines also consider whether otherwise objectionable mergers may be necessary to achieve efficiencies. The merging parties bear the burden of proof concerning efficiencies; the DOJ will only consider efficiencies that are merger-specific (i.e. could not be otherwise accomplished) and cognizable.

f. Failing Firm: There is a limited defense for failing firms. The defense is available if impending failure would cause the assets of one party to leave the market if the merger does not occur. This is a complete defense, and therefore (according to McChesney) should be considered first.

2. Pre-Merger Notification: The Hart-Scott-Rodino Act requires merging firms to notify the government of their deals. More importantly, it established a waiting period before larger mergers could be consummated. This has converted antitrust merger enforcement into a regime of bureaucratic regulation rather than litigation.

C. MODERN MERGER CASES

1. Approach of Lower Courts: Although courts recognize that the Supreme Court decisions from the 1960s permit the condemnation of mergers merely by reference to modest concentration increases, most judges have declined to rely on structural criteria alone. In fact, most courts refer to the Merger Guidelines when determining whether a merger violates the antitrust laws.

2. Hospital Corp. of America v. FTC (7th Cir. 1986): Judge Posner affirmed the decision of the FTC because it was not clearly erroneous; but it is obvious from the opinion that Posner disagrees with the decision. Defendant owned one

47

Page 48: Antitrust McChesney

hospital in Chattanooga, and wanted to acquire two additional hospitals; defendant also managed two other hospitals in Chattanooga. The acquisition would raise defendant’s market share to 26 percent in a highly concentrated market. Because the market was so highly concentrated, the merger would make it easier for firms to collude; because there was little cross-elasticity of demand, a rise in price will not significantly effect quantities sold to consumers; and because entry was difficult, it was unlikely that the market would check any increase in price. Thus, Posner concludes, there are facts supporting the finding that the merger might have anticompetitive effects. The efficiencies defendant claimed would result of the merger were not of the sort considered in merger analysis.

VII. SPECIAL RULES OF ANTITRUST: STANDING AND ANTITRUST INJURY

A. ANTITRUST “STANDING”1. Introduction: Several related concepts are often referred to as “antitrust

standing”: (1) purchaser rules; (2) antitrust injury; and (3) efficient enforcer. These are distinct requirements necessary for a private plaintiff to bring suit under the antitrust laws. They are a significant barrier to enforcement of the antitrust laws by “private attorneys general.”

2. Illinois Brick v. Illinois (1977): The plaintiff was the state of Illinois, which claimed it had been injured by a price fixing conspiracy among producers of concrete blocks. The defendants sold their product to masonry contractors, who submit their bids to general contractors, who in turn submit their bids to customers such as the state. Thus, plaintiff was not the direct purchaser of the product; the injury had therefore been passed down the chain of supply. The Court held that indirect purchasers did not have standing to sue for antitrust violations at the manufacturing level. This was based in large part on a prior decision in which the Court had proscribed defendants from using “passing on” as a defense against direct purchasers who passed the loss to consumers; the Court reasoned that it would be unfair to allow passing on to be used offensively against the manufacturer. The Court also feared that permitting indirect purchaser to sue would create intolerable administrative difficulties as courts sought to trace the amount and locus of harm throughout the distribution chain and apportion damages to each claimant.

B. ANTITRUST INJURY

1. In General: In Brunswick Corp. v. Pueblo Bowl-O-Mat, Inc. (1977), the Court established the requirement that the private plaintiff show that its injury resulted from the anticompetitive effects of the defendant’s conduct. The Court emphasized that the asserted injury must flow from a reduction or injury to competition—plaintiffs must prove injury “of the type the antitrust laws were intended to prevent and that flows from that which makes defendants’ acts unlawful.

2. Cargill v. Monfort of Colorado, Inc. (1986): This was a private challenge to a proposed merger in which plaintiff sought an injunction rather than damages. The question was whether, in this situation, plaintiff was required to demonstrate antitrust injury. The Court held that injunctive relief also requires a

48

Page 49: Antitrust McChesney

plaintiff to show antitrust injury. Furthermore, plaintiff only demonstrated that the merger might harm it because defendant might engage in predatory pricing, thereby depriving plaintiff of potential profits and possibly driving it out of business. The Court noted that, even though the government might have prevailed if it had brought suit, plaintiff could not prevail because it was basically saying that the increased competition would harm its business; this is not the type of harm the antitrust law seeks to redress, and thus plaintiff failed to show antitrust injury.

3. Atlantic Richfield Co. v. USA Petroleum Co. (1990): The question was whether a plaintiff must demonstrate antitrust injury when the defendant’s acts are per se illegal under the antitrust laws. Plaintiff alleged that defendant was engaged in vertical, maximum price fixing with its distributors (still subject to the per se rule of Albrecht in 1990). The Court held that this did not give rise to antitrust injury to a competitor—the rule against maximum price fixing is to redress the harm to consumers and dealers, not competitors; indeed, competitors might actually benefit from the pricing policy. “When a firm lowers prices but maintains them above predatory levels, the business lost by rivals cannot be viewed as an ‘anticompetitive’ consequence of the claimed violation. A firm complaining about the harm it suffers from nonpredatory price competition is really claiming that it is unable to raise prices. This is not antitrust injury; indeed, cutting prices in order to increase business often is the very essence of competition.” The significance of this case is that it makes it very difficult for competitors to sue for antitrust violations of its rivals: if prices are too low, then (as in this case) there is no antitrust injury; if prices are too high, there is no actual injury.

49


Recommended